Cutaneous Manifestations of Systemic Disease Flashcards

1
Q

1- The patient was recently hospitalized for a spontaneous pneumothorax. What is the underlying genetic mutation?

A. FLCN
B. CYLD
C. MSH2
D. PTEN
E. Menin

A

Correct choice: A. FLCN

Explanation: The figure illustrates fibrofolliculomas seen in Birt-Hogg-Dube syndrome. The syndrome features autosomal dominant inheritance, multiple fibrofolliculomas, trichodiscomas, lipomas, oral fibromas, renal cell carcinoma, medullary thyroid carcinoma, and colon cancer. Patients are at risk for pulmonary cysts and spontaneous pneumothoraces. The association mutation is FLCN (folliculin gene). The other mutations are found in Familial cylindromatosis (CYLD), Muir-Torre (MSH2), Cowden/PTEN hamartoma syndrome and Bannayan-Riley-Ruvalcaba syndrome (PTEN), and Multiple endocrine neoplasia (MEN) type I or Werner syndrome (Menin).

How well did you know this?
1
Not at all
2
3
4
5
Perfectly
2
Q

2- Which of the following is most commonly associated with hepatitis C infection?

A. Essential mixed cryoglobulinemia
B. Rheumatoid arthritis
C. Relapsing polychondritis
D. Wegener’s granulomatosis
E. Dermatomyositis

A

Correct choice: A. Essential mixed cryoglobulinemia

Explanation: Hepatitis C infections can present as urticaria or papable purpura and cryoglobulinemia. Studies in patients with mixed cryoglobulinemia syndrome found that 95% had signs of HCV infection.

How well did you know this?
1
Not at all
2
3
4
5
Perfectly
3
Q

3- A 35 year old man with a history of celiac disease presents with a beefy, red tongue, hyperpigmented palmar creases, and premature grey hair. Which of the following statements is correct?

A. The best therapy is riboflavin 5mg/day
B. This condition mimics folate deficiency
C. It is often associated with carcinoid tumors which divert tryptophan to serotonin
D. This condition can be caused by azithioprine, 5-FU, and isoniazid
E. Eating raw egg whites is a risk factor

A

Correct choice: B. This condition mimics folate deficiency

Explanation: This condition is vitamin B12 deficiency and is characterized by glossitis and hyperpigmentation in sun exposed areas and creases. Neurologic abnormaolities and megaloblastic anemia can be seen. The symptoms can mimic folic acid deficiency. Riboflavin (B2) is associated with oral-ocular-genital syndrome. Carcinoid tumors as well as azathioprine, 5-FU, and isoniazid are associated with niacin deficiency. Eating raw eggs is a risk factor for biotin deficiency.

How well did you know this?
1
Not at all
2
3
4
5
Perfectly
4
Q

4- Which of the following is FALSE regarding Fabry’s disease?

A. It is inherited in an X-linked recessive fashion
B. It is associated with acral parasthesias
C. It is inherited in an X-linked dominant fashion
D. It may be associated with renal failure
E. The etiology is a defect in alpha-galactosidase A

A

Correct choice: C. It is inherited in an X-linked dominant fashion

Explanation: Fabry’s disease is an X-linked lysosomal disorder that leads to excessive deposition of neutral glycosphingolipids in the vascular endothelium. The disorder is caused by a deficiency of alpha-galactosidase-A leading to progressive endothelial accumulation of glycosphingolipids. This accumulation accounts for the associated clinical abnormalities of skin, eye, kidney, heart, brain, and peripheral nervous system. Acroparesthesias are a frequent presenting symptom of Fabry’s disease. The primary cutaneous manifestation are angiokeratomata (angiokeratoma corporis diffusum), which are most common in a “bathing-trunk” distribution. Lens opacities and retinal and conjunctival vascular malformations may be found in the eyes. Patients may experience cardiac disease, stroke or renal failure.

How well did you know this?
1
Not at all
2
3
4
5
Perfectly
5
Q

5- You are called to evaluate a patient with these cutaneous findings. Once the diagnosis is confirmed, which additional test is indicated in the work-up for underlying causes?

A. Colonoscopy
B. Blood cultures
C. Hemoglobin A1C
D. Fat pad biopsy
E. Hepatitis C Antibody

A

Correct choice: E. Hepatitis C Antibody

Explanation: Porphyria cutanea tarda presents with vesicles and erosions with milia and scarring on sun-exposed skin, classically the dorsal hands. There is a deficiency in uroporphyrinogen decarboxylase (UD) and urine uroporphyrinogen is elevated. The most common associated conditions are Hepatitis C, HIV, iron overload states (hemochromatosis) and environmental triggers (alcohol, estrogens, polyhalogenated hydrocarbons). Treatment includes phlebotomy and low dose oral antimalarials.

How well did you know this?
1
Not at all
2
3
4
5
Perfectly
6
Q

6- Paraproteinemia is associated with all EXCEPT:

A. Scleromyxedema
B. Necrobiotic xanthogranuloma
C. Scleredema
D. Sclerosing panniculitis
E. Plane xanthoma

A

Correct choice: D. Sclerosing panniculitis

Explanation: Sclerosing panniculitis (lipodermatosclerosis) displays characteristic changes in the fat (lipomembranous change); it is not associated with paraproteinemia. Generalized plane xanthomas, scleromyxedema, necrobiotic xanthogranuloma, scleredema, erythema elevatum diutinum, xanthoma disseminatum, and pyoderma gangrenosum have all been associated with a paraproteinemia.

How well did you know this?
1
Not at all
2
3
4
5
Perfectly
7
Q

7- A patient with multiple deeply pigmented papules has a skin biopsy which reveals an epitheloid blue nevus. The next appropriate step is:

A. Reassure the patient and follow up as needed
B. Schedule prophylactic excision of the lesion
C. Begin a malignancy work-up
D. Refer to genetics
E. Order an echocardiogram

A

Correct choice: E. Order an echocardiogram

Explanation: Epithelioid blue nevi have been reported with and without association with cardiac myxomas as a component of the Carney complex (NAME/LAMB syndrome). Carney complex is an autosomal dominant disorder caused by mutations in PRKAR1A. Patients have cutaneous and atrial myxomas, blue nevi, ephelides, adrenocortical disease, and testicular tumors. The other options are not as appropriate as ordering an echocardiogram.

How well did you know this?
1
Not at all
2
3
4
5
Perfectly
8
Q

8- A homeless patient presents with a scaling, pustular periorifical eruption around the mouth and genitalia. What lab abnormality is associated with this condition?

A. Increased zinc level
B. Increased copper level
C. Decreased copper level
D. Decreased alkaline phosphatase level
E. Increased alkaline phosphatase level

A

Correct choice: D. Decreased alkaline phosphatase level

Explanation: In a homeless patient with poor nutrition, acrodermatitis enteropathica due to zinc deficiency can present with a scaling eruption of the periorificial regions. In addition to a low zinc level, levels of alkaline phosphatase, a zinc dependent enzyme, are decreased. Remaining choices are incorrect as they do not capture the lab abnormality of acrodermatitis enteropathica.

How well did you know this?
1
Not at all
2
3
4
5
Perfectly
9
Q

9- What is the most likely associated chronic infection?

A. Hepatitis B
B. Hepatitis C
C. HIV
D. HTLV-1
E. Mycobaterium Tuberculosis

A

Correct choice: B. Hepatitis C
Explanation: This is an example of necrolytic acral erythema, which is associated with chronic hepatitis C infection. Hepatitis B is associated with classic PAN, among other conditions. HIV is associated with many dermatoses. HTLV-1 is associated with mycosis fungoides. TB can be

associated with Erythema Induratum and tuberculid reactions. None of the above conditions is classically associated with necrolytic acral erythema.

How well did you know this?
1
Not at all
2
3
4
5
Perfectly
10
Q

10- A patient with gastric cancer develops acanthosis nigricans and a sudden eruption of numerous warty stuck-on papules on the trunk. What other finding may be seen?

A. Thickened, velvety palms with pronounced dermatoglyphics
B. Follicular spicules on the nose
C. Carpal tunnel syndrome
D. Periorbital pupura
E. Migratory thrombophlebitis

A

Correct choice: A. Thickened, velvety palms with pronounced dermatoglyphics

Explanation: Thickened, velvety palms with pronounced dermatoglyphics describes tripe palms (aka acanthosis palmaris). Tripe palms without acanthosis nigricans is associated with underlying lung cancer, whereas tripe palms with acanthosis nigricans is associated with underlying gastric cancer. Follicular spicules on the nose may be seen in multiple myeloma. Carpal tunnel syndrome and periorbital purpura can be seen in primary systemic amyloidosis (which can be associated with multiple myeloma). Migratory thrombophlebitis is associated with pancreatic cancer.

How well did you know this?
1
Not at all
2
3
4
5
Perfectly
11
Q

11- A 39 year old female has noticed a darkening area on both of her ears over the course of several decades. She denies supplement or medication intake and she does not use any topical products; otherwise she endorses a history of joint pains and stiffness. On physical exam, she has darkening of the anti-helix of both ears. What is the most likely diagnosis?

A. Argyria

B. Exogenous ochronosis
C. Relapsing polychondritis
D. Alkaptonuria
E. Dermal filler injection complication

A

Correct choice: D. Alkaptonuria

Explanation: The patient has alkaptonuria. The ears are a common site of involvement and show visible darkening over time. The musculoskeletal system can be involved as well. The patient does not endorse a history of any supplemental intake, thus argyria from silver toxicity is unlikely. She denies the use of any topical products, making exogenous ochronosis unlikely (and this condition is more common on the face where hydroquinone is more likely to be applied). Dermal filler injection is not correct as the patient has no history of dermal filler injection and the ear would represent an atypical site for injection of dermal filler. Relapsing polychondritis can present with musculoskeletal complaints, however, the ear and/or nose is typically inflamed and painful during episodes - our patient notices progressive darkening of the ear over time in contrast.

How well did you know this?
1
Not at all
2
3
4
5
Perfectly
12
Q

12- A 61-year-old woman presents with extensive erosions with hemorrhagic crusting of the lips, as well as several inflammatory erosions on the upper trunk. A peri-lesional biopsy for DIF shows deposition of C3 and IgG intercellularly and along the BMZ. Which of the following benign neoplasms is the most likely cause of this woman’s mucocutaneous lesions?

A. Thymoma
B. Uterine leiomyoma
C. Teratoma
D. Hepatic adenoma
E. Castleman’s disease

A

Correct choice: E. Castleman’s disease

Explanation: The stem describes the clinico-path findings seen in paraneoplastic pemphigus (PNP), which can be associated with both benign or malignant neoplasms. Among benign neoplasms, Castleman’s disease and thymoma have been reported as causes (Castleman’s disease more commonly than thymoma). Thymoma is less likely than Castleman’s disease to be the cause. The remaining answer choices are not associated with PNP.

How well did you know this?
1
Not at all
2
3
4
5
Perfectly
13
Q

13- A 71-year-old man presents to your clinic for a new asymptomatic, red-purple rash around his eyes. You notice that the lesions do not blanch with pressure. Which of the following is the most likely underlying malignancy?

A. Lung adenocarcinoma
B. Gastric adenocarcinoma
C. Multiple myeloma
D. Small cell lung cancer
E. Acute myelogenous leukemia (AML)

A

Correct choice: C. Multiple myeloma

Explanation: This patient’s presentation is concerning for periorbital “pinch” purpura, which occurs classically in primary systemic amyloidosis. Up to 25% of cases of primary systemic amyloidosis are due to multiple myeloma. Thus, this patient should undergo SPEP, UPEP, and serum immunofixation electrophoresis testing to evaluate for myeloma. The remaining listed malignancies are not typically associated with primary systemic amyloidosis.

How well did you know this?
1
Not at all
2
3
4
5
Perfectly
14
Q

14- Which of the following findings can be elicited during examination of these widespread skin lesions?

A. Buttonhole sign
B. Dimple sign
C. Crowe’s sign
D. Darier’s sign
E. Pseudo-Darier’s sign

A

Correct choice: A. Buttonhole sign

Explanation:With application of gentle pressure, a neurofibroma will easily invaginate into the subcutis, a finding known as the buttonhole sign. This question asks the examinee to identify neurofibromas by multiple soft, rubbery, skin colored to pink-tan papulonodules, and apply their knowledge of classic skin physical exam signs. When pressed on gently, a neurofibroma will invaginate into the subcutis, demonstrating the so-called buttonhole sign (choice 1). The dimple sign (choice 2) refers to the downward movement of a dermatofibroma when it is pinched. Crowe’s sign (choice 3) is axillary freckling seen in neurofibromatosis type I; it is not a feature of the neurofibromas themselves. Darier’s sign (choice 4) is local urtication with rubbing of lesions of cutaneous mastocytosis, while the pseudo-Darier’s sign (choice 5) is transient induration occurring with rubbing of a smooth muscle hamartoma.

How well did you know this?
1
Not at all
2
3
4
5
Perfectly
15
Q

15- this patient likely has which of the fellowing complaints in addition to the photo shown?

A- Hematuria
B- Dysphagia
C- proximal muscle weakness
D- Numerous Telangiectasia
E- Immunodeficiency

A

Correct choice: C-proximal muscle weakness

Explanation: Ragged cuticle is classically seen in dermatomyositis ,but can be seen in other autoimmune connective tissue diseases.

How well did you know this?
1
Not at all
2
3
4
5
Perfectly
16
Q

16- A patient presented with sudden onset numerous waxy brown papules on the trunk Which malignancy you should be most concerned with ?

A- lung carcinoma
B- Adenocarcinoma of GI tract
C- malignant melanoma
D- Non- Hodgkin lymphoma
E- Hodgkin lymphomas

A

Correct choice: B- Adenocarcinoma of GI tract

Explanation: The photo represents the sign of leser trelat which is the most commonly associated with GIT adenocarcinoma .this has also seen in lymphoma, breast ,cancer ,lung cancer.

How well did you know this?
1
Not at all
2
3
4
5
Perfectly
17
Q

17- A patient with history of AML was admitted for induction therapy with cytarabine and danurubicin .2 weeks later, she developed the fellowing painful rash .what is the diagnosis?

A- leukemia cutis
B- erythema elevatum diutinum
C- cystic acne
D- neutrophilic eccrine hidradenitis
E- Sarcoidosis

A

Correct choice: D - neutrophilic eccrine hidradenitis

How well did you know this?
1
Not at all
2
3
4
5
Perfectly
18
Q

Based on the primary cell type involved in pyoderma gangrenosum, which of the following treatments would be appropriate?

A. Acitretin
B. Crisaborole
C. Dapsone
D. Dupilumab
E. Apremilast

A

Correct choice: C. Dapsone

Explanation: Pyoderma gangrenosum (PG) is a neutrophil driven disease process. In addition to its usage as an antibiotic, dapsone is anti-inflammatory and has anti-neutrophilic properties. Topical and/or systemic dapsone preparations can be appropriate choices for the treatment of PG.

How well did you know this?
1
Not at all
2
3
4
5
Perfectly
19
Q

19- A biopsy from a systemically ill patient with porcelain white macules and papules on the trunk and extremities shows wedge-shaped necrosis stemming from a deep vessel. Involvement in which of the following systems most commonly is responsible for a fatal outcome?

A. Cardiac
B. Gastrointestinal
C. Renal
D. Endocrinologic
E. Neurologic

A

Correct choice: B. Gastrointestinal
Explanation: The correct answer is B. The question describes Degos disease (malignant atrophic papulosis). Although neurological involvement is common, GI involvement more commonly results in a fatal outcome (often perforation after necrosis). Neurologic involvement can also occur as headache, hemiparesis, aphasia, and cranial nerve involvement, but is less likely to cause fatality.

How well did you know this?
1
Not at all
2
3
4
5
Perfectly
20
Q

20- This patient likely has which of the following complaints in addition to the photo shown?

A. Proximal muscle weakness
B. Immunodeficiency
C. Numerous telangiectasias
D. Hematuria
E. Dysphagia

A

Correct choice: A. Proximal muscle weakness

Explanation:. Ragged cuticles (Samitz sign) is classically seen in dermatomyositis, but can also be seen in other autoimmune connective tissue diseases.
The other features listed aren’t as commonly seen in dermatomyositis. Hematuria can be seen in HSP which has palpable purpuric papules and macules on extensor surfaces and buttocks. Dysphagia can be seen in systemic sclerosis & Plummer-Vinson syndrome. Numerous telangiectasias can occur in Hereditary Hemorrhagic Telangiectasia or Ataxia-Telangiectasia. Immunodeficiency can occur in Wiskott-Aldrich syndrome, Chronic Granulomatous disease, and Severe combined immunodeficiency.

How well did you know this?
1
Not at all
2
3
4
5
Perfectly
21
Q

21- If a patient had a history of recurrent ischemic strokes and presents with livedo reticularis, which of the following labs abnormalities would you most likely find, assuming it is clinically relevant?

A. Platelets of >800,000/microliter
B. Low levels of protein S
C. Anti-beta-2 glycoprotein antibodies
D. Anti-heparin antibodies
E. Factor V mutation

A

Correct choice: C. Anti-beta-2 glycoprotein antibodies

Explanation: The correct answer is C, which is one of 3 labs in the antiphospholipid antibody syndrome. The combination of livedo reticularis (or livedo racemosa) with neurological disease, especially ischemic strokes, points to Sneddon Syndrome, which may show a positive workup for antiphospholipid antibody syndrome.

How well did you know this?
1
Not at all
2
3
4
5
Perfectly
22
Q

22- A patient presents to clinic with a new onset of bullous lesions on his ankles. He notes immediately prior to the onset of these lesions, he was taking a walk through a local park. Which of the following is the most common association?

A. IgA gammopathy
B. Chronic Lymphocytic leukemia
C. History of scabies infestation
D. Thrombocytopenia
E. Exfoliative Toxin A

A

Correct choice: B. Chronic Lymphocytic leukemia

Explanation: This question stem is describing a bullous arthropod reaction. This is typically seen in the setting of CLL. IgA gammopathy could result in IgA pemphigus but the sudden nature of these lesions would suggest against this. Exfoliative Toxin A would be associated with bullous impetigo but the time course of the lesions starting immediately after a walk in the park would suggest against this dx. Acropustulosis of infancy has been associated with prior scabies infection. Thrombocytopenia has not been directly associated with bullous lesions.

How well did you know this?
1
Not at all
2
3
4
5
Perfectly
23
Q

23- A 65-year old male has long standing diabetes mellitus. Which of the following cutaneous lesions are most likely to develop in this patient?

A. Rapidly developing tense blisters on the hands and feet
B. Sudden crops of firm, yellowish papules with a rim of erythema
C. Bilateral red-brown atrophic macules on shins
D. Waxy skin over the dorsal hands and feet
E. Firm, annular erythematous plaques with central clearing

A

Correct choice: C. Bilateral red-brown atrophic macules on shins

Explanation: Diabetic Dermopathy, or “shin spots”, are the most common manifestation of diabetes mellitus. It presents as brown atrophic macules and patches on the shins. These occur in approximately 50% of diabetics and are due to microangiography. These patients are at risk for other vasculopathy-associated manifestations such as nephropathy, neuropathy, and retinopathy. Unfortunately, there are no effective treatments for the condition. The other answer choices represent less common cutaneous manifestations of diabetes, including granuloma annulare (firm, annular erythematous plaques with central clearing), eruptive xanthomas (sudden crops of firm, yellowish papules with a rim of erythema), bullosis diabeticorum (rapidly developing tense blisters on the hands and feet), and scleroderma-like skin (waxy skin over the dorsal hands and feet).

How well did you know this?
1
Not at all
2
3
4
5
Perfectly
24
Q

24- An 80 year old woman presents to your office with new-onset, yellow, thin, smooth plaques around her eyes, body folds, and trunk. Which of the following underlying conditions is associated with this presentation?

A. Acute myelogenous leukemia (AML)
B. Multiple myeloma
C. Gastric adenocarcinoma
D. Castleman’s disease
E. Thymoma

A

Correct choice: B. Multiple myeloma

Explanation: Diffuse normolipemic plane xanthoma (DNPX) was first described by Altman and Winkelmann in 1962. It is a rare and non-inherited form of xanthomatosis. Clinically, the dermatosis is characterized by the presence of symmetric yellowish-orange plaques that favor the neck, upper trunk, flexural folds and periorbital region. It has been recognized to be associated with hematological diseases, especially with multiple myeloma and monoclonal gammopathy. Thymoma and Castleman’s disease are associated with paraneoplastic pemphigus. Gastric adenocarcinoma is associated with acanthosis nigricans and tripe palms, as well as the suddent onset of multiple seborrheic keratoses (Sign of Leser-Trelat). AML is associated with Sweet’s syndrome.

How well did you know this?
1
Not at all
2
3
4
5
Perfectly
25
Q

25- A patient is referred for these findings. Which study is the most appropriate in this setting?

A. Ceruloplasmin
B. Arsenic level
C. Iron
D. Chest X-ray
E. BUN/Cr

A

Correct choice: A. Ceruloplasmin

Explanation: This kodachrome is of blue lunulae, which can occur with zidovudine, silver, or Wilson’s disease. The test for Wilson’s disease would be decreased ceruloplasmin. It is caused by an autosomal recessive ATP7B gene defect (copper-transporting ATPase). It causes an accumulation of copper, resulting in blue lunulae, Kayser-Fleishcher rings, neurologic sequelae, and liver failure. BUN/Cr would evaluate for CKD in half and half nails. Chest X-ray could be used for evaluating pulmonary disease in clubbing. Iron deficiency anemia can cause koilonychia. Arsenic would cause Mee’s lines which would present with transverse white lines.

How well did you know this?
1
Not at all
2
3
4
5
Perfectly
26
Q

26- A 22 y/o patient presents with the following dermatologic finding. A biopsy is performed with the below findings. Which of the following is the appropriate first step?

A. Check ASO titer

B. Check platelet count
C. Check HgbA1C
D. Excision of lesion
E. Check TSH

A

Correct choice: A. Check ASO titer

Explanation: This kodachrome and histologic photos show erythema nodosum. Clinically, it presents with erythematous, tender nonulcerating nodules that progress to a violaceous bruise-like color. Histologically it presents with a septal panniculities; one would see septal edema, a mixed inflammatory infiltrate, and widened septa with fibrosis.
Streptococcus is the most common infectious cause of erythema nodosum (EN) and therefore ASO titers should be evaluated as a first line diagnostic test for EN. An evaluation of other infectious etiologies, a medication review, and a thorough personal and family history for rheumatologic illnesses should be taken. Excision of erythema nodosum would be an inappropriate first choice, as spontaneous resolution of EN typically occurs within 3-6 weeks. If bruising is suspected, platelet count may be evaluated for thrombocytopenia. However, the pathologic image here indicates a septal panniculitis, which would not occur in thrombocytopenia. HgbA1c could be important if necrobiosis lipoidica diabeticorum is suspected. This would present with well demarcated, firm, waxy, yellowish brown plaques, which is not seen in this photo. TSH would not be the first step in the work-up for erythema nodosum.

How well did you know this?
1
Not at all
2
3
4
5
Perfectly
27
Q

27- A patient presents with thinning of the lateral third of her eyebrows. Which of the following is not associated with this finding?

A. Hyperthyroidism
B. Mycosis fungoides
C. Syphillis
D. Trichotillomania
E. Leprosy

A

Correct choice: A. Hyperthyroidism

Explanation:The sign of Hertoghe or Queen Anne’s sign is a thinning or loss of the outer third of the eyebrows. It is typically seen in HYPOthyroidism not HYPERthyroidism. Hertoghe’s sign can be

seen in Ulerythema ophroygenes, hypothyroidism, leprosy, alopecia mucinosa, folliculotrophic MF, ectodermal dysplasia, trichotillomania, and syphilis.

How well did you know this?
1
Not at all
2
3
4
5
Perfectly
28
Q

28- Hyperkeratotic follicular nasal papules have been described as a paraneoplastic sign in the setting of which neoplasm?

A. Multiple myeloma
B. Castleman’s tumor
C. AML
D. Adenocarcinoma of the lung
E. Renal cell carcinoma

A

►A

Hyperkeratotic follicular nasal papules have been described as a paraneoplastic phenomenon in the setting of multiple myeloma. AML is associated with Sweet’s syndrome and Castleman’s tumor is associated with paraneoplastic pemphigus.

How well did you know this?
1
Not at all
2
3
4
5
Perfectly
29
Q

29 - Which of the following may be associated with Graves‟ disease?
A. Dermatitis herpetiformis
B. Geographic tongue
C. Hypohidrosis
D. Madarosis
E. Thick, pale lips

A

►A

Graves‟ disease is a thyrotoxic condition that results from the production of thyroid-stimulating immunoglobulins (TSI) by stimulated B lymphocytes. The TSI bind to the thyroid-stimulating hormone (TSH) receptor and mimic TSH thereby stimulating thyroid growth and thyroid hormone overproduction. Signs and symptoms of Graves‟ disease include goiter, tachycardia, exophthalmos, tremor, sweating, palpitations, smooth moist skin, diarrhea, sleeplessness, irritability, and weight loss. Autoimmune cutaneous disease may also be associated with Graves‟

disease including vitiligo, dermatitis herpetiformis, herpes gestationis, and pemphigus vulgaris. Cutaneous manifestations of hypothyroidism include xerosis, hyperhidrosis, yellowish hue, myxedema, and purpura. The hair may be dry, brittle and coarse; alopecia may be diffuse and/or involve the lateral eyebrow (madarosis).

How well did you know this?
1
Not at all
2
3
4
5
Perfectly
30
Q

30 -A patient with gluten-sensitive enteropathy presents with vesicles on the extensor surfaces of the extremities. What findings are most likely on a perilesional biopsy?

A. Granular IgA at the dermoepidermal junction on direct immunofluorescence
B. Linear C3 and IgG at the dermoepidermal junction on direct immunofluorescence
C. Linear IgA surrounding vessels on direct immunofluorescence
D. Pautrier”s micro-abscesses in the epidermis on H & E
E. Leukocytoclastic vasculitis

A

►A

This patient has dermatitis herpetiformis (DH). DH is characterized by itchy papulovesicles on extensor surfaces of the extremities. Neutrophilic infiltrates at the dermal papillae with vesicle formation are found on histopathologic examination. On immunofluorescence, granular IgA (directed against transglutaminase) deposits are found in perilesional skin. Over 90% of patients have gluten-sensitive enteropathy of varying severity. Dapsone is almost universally therapeutic for the skin disease, but not the enteropathy. A gluten free diet, although difficult to m aintain, treats the enteropathy. The majority of DH patients have the HLA class II DQ2 genotype. On indirect immunofluorescence, one may find antigliadin, antiendomyseal or antireticulin antibodies. Autoimmune thyroid disease (especially Hashimoto”s thyroiditis), enteropathyassociated T-cell non-Hodgkin”s lymphoma and insulin-dependent diabetes are the most common autoimmune associations. Pautrier”s micro-abscesses are found in mycosis fungoides. Leukocytoclastic vasculitis is not a feature of DH.

How well did you know this?
1
Not at all
2
3
4
5
Perfectly
31
Q

31- A 64-year-old woman presents with lichenified keratotic plaques on the bilateral lower extremities. Biopsy reveals suppurative inflammation and collagen extending from the dermis through the epidermis. Which is the most likely associated systemic disease:

A. Congestive Heart Failure

B. Hepatitis C Infection
C. Hypothyroidism
D. Chronic kidney disease
E. Castleman‟s disease

A

►D

Chronic kidney disease is associated with perforating disorders. These disorder are characterized by pruritus and a biopsy revealing collagen and inflammatory cells broaching the epidermis. The other disorders are not closely associated with perforating diseases.

How well did you know this?
1
Not at all
2
3
4
5
Perfectly
32
Q

32- A patient presents with mild mental retardation, infertility, joint contractures, short stature, ichthyosis, and sparse hair with trichoschisis. All of the following are true regarding this patient’s condition EXCEPT:

A. This syndrome is inherited in an autosomal recessive manner
B. If photosensitivity is a feature, gonad size is likely normal
C. Patients may have associated cataracts
D. The syndrome is caused by impaired nucleotide excision repair
E. Perifoveal glistening white dots are a feature

A

►E

The patient described has IBIDS syndrome (Ichthyosis, Brittle hair, Intellectual impairment, Decreased fertility, and Short stature), or Tay’s syndrome. This syndrome is autosomal recessive, caused by mutations in the ERCC2/XPD or ERCC3/XPB genes, resulting in impaired nucleotide excision repair. Cataracts may be a feature. If photosensitivity is a feature (PIBIDS), gonal size is likely normal. Perifoveal glistening white dots are a feature of Sjorgen-Larsson Syndrome, not IBIDS.

How well did you know this?
1
Not at all
2
3
4
5
Perfectly
33
Q

33- A patient with congenital hypertrophy of retinal epithelium is most likely to have:

A. An autosomal dominant mutation in the MSH2 gene

B. Pheochromocytoma
C. Adenomatous polyposis
D. Tram-track calcifications on head radiograph
E. Peg-shaped teeth

A

►C

Congenital hyperpigmentation of the retinal pigment (CHRPE) is an early feature of Gardner syndrome (GS). It is found in approximately 60% of patients with GS. GS is an autosomal dominant disorder characterized by precancerous intestinal polyposis and subsequent adenocarcinoma of the gastrointestinal tract. Cutaneous manifestations include epidermoid cysts, osteomas, desmoids and fibrous tumors. A mutation in the adenomatous polyposis coli (APC) gene, a tumor suppressor gene, is responsible for the disease. Most patients develop colon carcinoma by the 2nd or 3rd decade. Therefore, prophylactic colectomy is warranted. Mutations in the MSH2 gene are found in Muir-Torre syndrome. Pheochromocytomas are found in multiple endocrine neoplasia (MEN) syndromes IIa and IIb. Tram track calcifications are found in SturgeWeber syndrome. Peg-shaped teeth are found in multiple syndromes including ectodermal dysplasia.

How well did you know this?
1
Not at all
2
3
4
5
Perfectly
34
Q

34- A 14-year-old female presents with gastrointestinal pain and seizures. Laboratory studies reveal hyponatremia. Which of the following medications was most likely to induce her systemic symptoms:

A. Phenytoin
B. Diphenhydramine
C. Griseofulvin
D. Morphine
E. Acetaminophen

A

►C

Acute intermittent porphyria (AIP) is associated with colicky abdominal pain, neurologic symptoms, and SIADH. This disorder is caused by deficiency of porphobilinogen deaminase. AIP may be exacerbated by drugs including barbiturates, sulfonamides, and griseofulvin, among others.

How well did you know this?
1
Not at all
2
3
4
5
Perfectly
35
Q

35- Which of the following is not a feature of Cronkhite-Canada syndrome?

A. Diarrhea
B. Alopecia
C. Lung carcinoma
D. Dystrophic nails
E. Hyperpigmented macules

A

►C

Cronkhite-Canada syndrome is a rare, non-familial disease characterized by patchy alopecia, nail changes or loss, lentigines, inflammatory polyps, abdominal pain, and a protein losing enteropathy.

How well did you know this?
1
Not at all
2
3
4
5
Perfectly
36
Q
  1. -Which of the following is a paraneoplastic disease most often associated with lung carcinoma?

A. Hypertrichosis lanuginosa acquisita
B. Dermatomyositis
C. Acanthosis nigricans
D. Paraneoplastic pemphigus
E. Erythroderma

A

►A

Hypertrichosis lanuginosa acquisita is the abrupt onset of downy, soft, non-pigmented hair of the face, trunk, and extremities. It may have an associated glossitis. It is associated with underlying lung carcinoma and may resolve with treatment of the underlying malignancy.

How well did you know this?
1
Not at all
2
3
4
5
Perfectly
37
Q
  1. Which of the following is true regarding piebaldism?

A. It is caused by a mutation in the GJB2 gene
B. It is caused by defective metabolism of phytanic acid
C. It is caused by a defect in a protein subunit of a kinase that activates NFkappaB
D. It is caused by a deficiency of fatty aldehyde dehydrogenase

E. It is caused by a mutation in the proto-oncogene c-KIT

A

►E

Piebaldism is caused by an autosomal dominant mutation of the proto-oncogene c-KIT which encodes tyrosine a tyrosine kinase receptor on melanocytes, preventing activation by steel factor.

How well did you know this?
1
Not at all
2
3
4
5
Perfectly
38
Q
  1. A- middle-aged gentleman who avoided healthcare dies suddenl y from gastrointestinal hemorrhage. Post-mortem examination reveals multiple soft blue compressible tumors on the trunk, arms, and tongue. Blue rubber bleb nevus syndrome is caused by a mutation in what gene?

A. VMCM1
B. ENG
C. VEGF
D. PTEN
E. RET

A

►A

Blue rubber bleb nevus syndrome is a rare sporadic or autosomal dominant disorder with soft compressible blue tumors on the trunk and arms. Nocturnal pain is characteristic. Gastrointestinal hemangiomas can cause hemorrhage. Mutations in the VMCM1=TEK=Tie -2 gene are reported. It is also knkow n as VMCM syndrome.

How well did you know this?
1
Not at all
2
3
4
5
Perfectly
39
Q

39- Which of the following statements regarding porphyrias is TRUE?

A. Elevated uroporphyrins are found in the red blood cells of hepatoerythropoietic porphyria
B. Delta aminolevulenic acid is the only oxidized porphyrin
C. Acute intermittent porphyria is the most common form of porphyria
D. Griseofulvin is safe for those with variegate porphyria
E. Plasma fluoresces at 410 nm in patients with variegate porphyria

A

►B

Acute intermittent porphyria (AIP) is the second most common porphyria and is caused by a deficiency in porphobilinogen (PBG) deaminase, which is located in the cytosol. Patients suffer from colicky pain, paralysis and psychiatric disorders. There are no specific skin manifestations. PBG and aminolevulenic acid (ALA) are elevated in the urine. Attacks are precipitated by medications such as barbiturates, estrogen, griseofulvin, and sulfonamides as well as starvation, fever and infection. Treatment includes glucose loading and hematin infusion.

Congenital erythropoietic porphyria (CEP) or Günter‟s disease is caused by a defect in uroporphyrinogen III synthase, which is found in the cytosol. Patients are extremely photosensitive and erythema, blistering and scarring result. Patients present with red urine early in life along with hypertrichosis and red-stained teeth that fluoresce. Uroporphyrins (URO) are much high than coproporphyrins (COPRO) in the urine. URO is found in the red blood cells (rbcs) and COPRO is found in the stool. The rbcs display stable fluorescence.

Porphyria cutanea tarda (PCT) is the most common porphyria and is caused by a deficiency (usually sporadic) in uroporphyrinogen decarboxylase, which is found in the cytosol. Patients present with photosensitivity and blistering of sun-exposed areas, especially the dorsal hands. Hypertrichosis and sclerodermoid changes may occur as well. Liver disease (hepatitis C or alcoholic cirrhosis) is often present and hemochromatosis may be associated. Urine may fluoresce pink or coral-red with Wood‟s lamp. URO>COPRO in the urine and low levels of COPRO are found in the stool. Treatments include phlebotomy, antimalarials, and therapy for liver disease if appropriate.

Hereditary coproporphyria (HCP) is caused by a deficiency in coproporphyrinogen oxidase, which is found in the mitochondria. One-third of patients are photosensitive, and patients suffer gastrointestinal and neurological symptoms similar to AIP. Urine COPRO is elevated only with attacks, and COPRO is present in the stool.

Variegate porphyria is the result of decreased activity of protoporphyrinogen oxidase, which is present in the mitochondria. It combines the skin lesions of PCT with the systemic manifestations of AIP. Urine COPRO:URO is 1:1 or COPRO> URO to distinguish it from PCT, and PROTO is found in the stool. The plasma fluoresces at 626nm. Precipitators and treatments are similar to AIP.

Erythropoietic protoporphyria (EPP) is caused by ferrochetalase deficiency, which is present in the mitochondria. Patients experience immediate burning of the skin with sun exposure. Protoporphyrin IX, the only oxidized porphyrin in the heme pathway and absorbs in the Soret band (400-410nm). Patients have erythematous plaques in a photo-distribution. Urine porphyrins are normal. PROTO is

found in the rbcs and the stool. Excessive porphyrins deposited in the liver lead to gallstones and cirrhosis. Beta carotene may helpful.

Hepatoerythropoietic porphyria (HEP) is essentially a homozygous form of PCT, with deficiency in uroporphyrinogen decarboxylase. It is clinically similar to CEP with red urine and hypertrichosis, vesicles and scarring of sun-exposed skin. URO is present in the urine and COPRO in the stool. PROTO is present in rbcs which distinguishes it from CEP, which was URO in rbcs.

How well did you know this?
1
Not at all
2
3
4
5
Perfectly
40
Q

40- The expected histology of a biopsy take from the lesion shown in the image would

A. Increased mucin
B. Schumann bodies
C. Granulomatous infiltrate localized to the papillary dermis
D. Elastic fiber degeneration
E. Collagen degeneration

A

►E

The histologic findings of necrobiosis lipodica diabeticorum are: dermal granulomatous inflammation in a horizontal pattern, collagen degeneration, and normal or atrophic epidermis.

How well did you know this?
1
Not at all
2
3
4
5
Perfectly
41
Q

41 -Squamous cell carcinoma is seen in which syndrome?

A. Gorlin syndrome
B. Rombo syndrome
C. Nicolau-Balus syndrome

D. Rasmussen syndrome
E. Bazex syndrome (Acrokeratosis paraneoplastica)

A

►E

Basex syndrome, also Acrokeratosis paraneoplastica, presents with symmetric erythematous, nearly violaceous, psoriasiform dermatoses of the hands, feet, ears and nose. The syndrome is nearly associated with an underlying malignancy, usually squamous cell carcinoma of the upper aerodigestive tract.

How well did you know this?
1
Not at all
2
3
4
5
Perfectly
42
Q

42- All of the following statements are true regarding this condition EXCEPT:
A. Diabetes or glucose intolerance is found in 20% of these patients
B. This condition may be associated with cutaneous anesthesia, hypohidrosis, and partial alopecia
C. There is no impact of tight glucose control on the likelihood of developing this condition
D. 0.3-3% of diabetics have this skin condition
E. This condition is associated with increased dermal mucin

A

►E

Necrobiosis Lipoidica Diabeticorum (NLD) is found in 0.3-3% of diabetics. Approximately 20% of NLD patients have diabetes or glucose intolerance. It presents with single or mul tiple redbrown papules which progress to sharply demarcated yellow-brown atrophic, telangiectatic plaques with violaceous, irregular borders; common sites include the shins. Cutaneous anesthesia, hypohidrosis, and partial alopecia can be found. Pathology shows palisading granulomas containing degenerating collagen (necrobiosis); with NO increase in dermal mucin. There is no impact of tight glucose control on the likelihood of developing NLD.

How well did you know this?
1
Not at all
2
3
4
5
Perfectly
43
Q

43- Which of the following is NOT associated with Hepatitis C disease?

A. Mixed cryoglobulinemia
B. PCT
C. Single-stranded RNA viridae
D. Leukocytoclastic vasculitis
E. Single-stranded DNA viridae

A

►E

Hepatitis C virus (HCV) is a single-stranded RNA virus that is a member of the flaviviridae family. Approximately 20-30% of patients develop symptoms with acute infection and 70% will progress to chronic disease. Porphyria cutanea tarda or PCT is associated with HCV in a substantial percentage of patients; in one study, antibodies to HCV were found in 82% of PCT patients. Up to 80% of mixed cryoglobulinemia (MC) cases are associated with HCV. The incidence of lichen planus in HCV patients varies from region to region (0.1-35%). The histopathology of MC lesions is leukocytoclastic vasculitis. Another relatively common association is polyarteritis nodosa (PAN) which is also related to hepatitis B infection. Finally, generalized pruritis is a common complaint of HCV patients.

How well did you know this?
1
Not at all
2
3
4
5
Perfectly
44
Q

44- All of the following are seen more commonly in Crohn’s disease than in ulcerative colitis EXCEPT:

A. Oral cobblestoning
B. Polyarteritis nodosa
C. Pyostomatitis vegetans
D. Perineal fistulas
E. Perineal fissures

A

►C

Pyoderma vegetans consists of vegetating plaques and vesicopustules of intertriginous areas than heal with hyperpigmentation. When the process involves mucosal surfaces it is called pyostomatitis vegetans. These processes are both associated with ulcerative colitis, not Crohn’s disease.

How well did you know this?
1
Not at all
2
3
4
5
Perfectly
45
Q

45- Patients with plexiform neuroma and NF I who also have JXG are at increased risk for developing:

A. Juvenile chronic myelogenous leukemia
B. Non-Hodgkin’s Lymphoma
C. Esophageal cancer
D. Breast cancer
E. Medullary carcinoma of the thyroid gland

A

►A

Patient with NF 1 who have multiple juvenile xanthogranulomas are at 20 times greater risk for developing juvenile chronic myelogenous leukemia.

How well did you know this?
1
Not at all
2
3
4
5
Perfectly
46
Q

-46. The peak sensitivity to prophyrins occurs at which wavelengths?

A. 220-290nm
B. 290-320nm
C. 320-400nm
D. 400-410nm
E. 410-450nm

A

►D

The Soret band (400-410 nm) is the portion of ultraviolet wavelengths at which most porphyrins are most sensitive.

How well did you know this?
1
Not at all
2
3
4
5
Perfectly
47
Q
  1. A patient presents with plane/palmar xanthomas. The most likely genetic disorder would be:

A. familial lipoprotein lipase deficiency
B. Familial hypertriglyceridemia
C. Familial hypercholesterolemia
D. Familial dysbetalipoproteinemia
E. Cerebrotendinous xantomatosis

A

►D

Patients with Type III hyperlipidemia have both elevated triglyceride levels and cholesterol levels in the plasma. A genetic basis for the primary disorder, familial dysbetalipoproteinemia, has been well established. These patients present as adults with premature atherosclerosis and xanthomas, particularly plane (palmar) xanthomas.

How well did you know this?
1
Not at all
2
3
4
5
Perfectly
48
Q

48 -A 70-year old female develops erythema with fine adherent scale on acral skin that progresses to keratoderma and eventually a more generalized psoriasiform dermatitis. What is the most likely underlying malignancy?

A. Squamous cell carcinoma of the larynx
B. Adenocarcinoma of the colon
C. Adenocarcinoma of the breast
D. Squamous cell carcinoma of the vagina
E. Medullary thyroid carcinoma

A

►A

Acrokeratosis Paraneoplastica (aka Bazex) is always associated with malignancy. Most commonly, the associated malignancy is a squamous cell carcinoma of the upper aerodigestive tract. The skin disease usually follows the course of the malignancy.

How well did you know this?
1
Not at all
2
3
4
5
Perfectly
49
Q

49- Patients with Werner‟s syndrome typically experience which of the following types of cardiac disease?
A. Hypertrophic cardiomyopathy
B. Aortic aneurysms
C. Premature atherosclerosis
D. Cardiomegaly
E. Mitral valve prolapse

A

►C

Werner‟s syndrome or Adult Progeria is caused by autosomal recessive mutations in WRN (Recql2) gene, which encodes DNA helicase. This defect leads to defects in DNA repair and replication. Patients prematurely age and essentially experience many diseases of aging early in childhood and teen years. They exhibit tight atrophic skin, relatively large heads for body size, leg ulcers and cannities. Patients experience early, accelerated atherosclerosis leading to death by myocardial infarction. In addition, type II diabetes, cataracts, osteoarthritis, osteoporosis and hypogonadism are features.

How well did you know this?
1
Not at all
2
3
4
5
Perfectly
50
Q

50- A patient with end stage renal disease undergoes an MRI examination with contrast. He subsequently develops woody indurated plaques on the extremities. Fatalities seen in nephrogenic systemic fibrosis are due to what underlying process?

A. Fibrosis of respiratory muscles
B. Fibrosis of cardiac smooth muscle
C. Fibrosis of the gastrointestinal tract
D. Fibrosis of cerebral arteries
E. Fibrosis of the liver

A

►A

Nephrogenic systemic fibrosis is a chronic fibrosing disorder seen most commonly in the setting of renal failure. There is a reported association also with gadolinium as contrast in an MRI. Patients present with woody indurated plaques or nodules primarily of the extremities, with occasional involvement of the trunk. Fulminant and fatal disease is rare, occurring in about 5% of cases. Death is due to impaired ventilation secondary to fibrosis of respiratory muscles.

How well did you know this?
1
Not at all
2
3
4
5
Perfectly
51
Q

51- Which of the following skin findings is most closely linked to hepatocellular carcinoma as a paraneoplastic syndrome?

A. Pityriasis lichenoides
B. Pityriasis alba
C. Pityriasis amiantacea
D. Pityriasis rotunda
E. Pityriasis rosea

A

►D

Pityriasis rotunda is an dermatosis that features characteristic discrete, circular, scaly, brown patches on the trunk and extremities. Pityriasis rotunda may be associated with systemic diseases in certain racially predisposed groups (blacks), and has been linked to hepatocellular carcinoma.

How well did you know this?
1
Not at all
2
3
4
5
Perfectly
52
Q

52 -A 20-year-old male develops an eruption of 100’s of red-brown yellowish papules with involvement of the mucous membranes. He has no lymphadenopathy. The most likely diagnosis is:

A. Montgomery’s syndrome
B. Benign cephalic histiocytosis
C. Rosai-Dorfman disease
D. Necrobiotic xanthogranuloma
E. Multicentric reticulohistiocytosis

A

►A

This patient has Montgomery’s syndrome, or xanthoma disseminatum, which is characterized as a benign normolipidemic but disfiguring condition that demonstrates an eruption of 100’s of redbrown yellowish papules and plaques that may involve the mucous membranes. These patients should be evaluated for diabetes insipidus. Benign cephalic histiocytosis shows brownish-yellow papules on the upper face. Rosai-Dorfman, or sinus histiocytosis with massive lymphadenopathy, features polymorphic papules and plaques in the first two decades with fever, increased ESR, and cervical lymphadenopathy. Necrobiotic xanthogranuloma is characterized by red-orange plaques that may ulcerate. Finally, multicentric reticulohistiocytosis features coral beading around the fingers and is associated with arthritis mutilans and malignancy.

How well did you know this?
1
Not at all
2
3
4
5
Perfectly
53
Q

53- Which of the following porphyria cutanea tarda associations has a direct relationship to the level of urine uroporphyrins?

A. Dystrophic calcifications
B. Estrogen levels
C. Sclerodermoid changes
D. RBC fluorescence
E. Hypertrichosis

A

►C

Connective tissue dystrophic calcifications, increased estrogen levels, hypertrichosis and sclerodermoid changes are all associated with porphyria cutanea tarda, but sclerodermoid changes are the only finding with a direct relationship to urine uroporphyrin levels. Urine fluorescence occurs in PCT, not RBC fluorescence.

How well did you know this?
1
Not at all
2
3
4
5
Perfectly
54
Q

54- A 55 year old patient presents with new onset brown macules on arms, legs, face and palms. She gives a 3 month history of diarrhea, abdominal cramps, weight loss and protein-losing enteropathy. The most likely diagnosis is:

A. Peutz-Jeghers syndrome
B. Cowden disease
C. Ulcerative colitis
D. Cronkhite-Canada syndrome
E. Plummer-Vinson syndrome

A

►D

Cronkhite-Canada is an aquired disease characterized by the development of polyps throughout the GI tract. Patients can present with hyperpigmented macules as well as the sequella of GI malabsorption. The malignant transformation of polyps can occur.

How well did you know this?
1
Not at all
2
3
4
5
Perfectly
55
Q

55- Which of the following diseases is caused by an enzymatic defect that occurs in the mitochondria?

A. Porphyria Cutanea Tarda
B. Acute Intermittent Porphyria
C. Congenital Erythropoeitic Porphyria
D. Erythropoeitic Protoporphyria
E. Hepatoerythropoetic Porphyria

A

►D

Defects in many of the enzymes involved in heme synthesis are responsible for porphyrias. The first and last three steps of the heme synthesis pathway occur in the mitochondria.

How well did you know this?
1
Not at all
2
3
4
5
Perfectly
56
Q

56 -Which one of the following clinical findings suggests the diagnosis of multiple myeloma?

A. Sudden eruption of seborrheic keratoses
B. Hyperkeratotic follicular papules on the nose
C. Erythema, vesicles, and erosions in periorifical and acral areas
D. Concentric erythematous rings with trailing scale on trunk and extremities
E. Tripe palms

A

►B

Hyperkeratotic follicular nasal papules are associated with multiple myeloma. A sudden increase in number and size of seborrheic keratoses is called the sign of Lesser-Trelat and can be seen in gastric or colon carcinoma. Erythema and vesiculation in periorificial and acral areas is characteristic of necrolytic migratory erythema most often associated with pancreatic cancer.
Concentric erythematous rings with trailing scale is a feature of erythema gyratum repens most commonly seen in lung cancer. Tripe palms with AN more likely gastric whereas without AN more likely lung.

How well did you know this?
1
Not at all
2
3
4
5
Perfectly
57
Q

-All of the following disorders have an increased risk of systemic malignancy except

A. Rothmund Thompson syndrome
B. Cockayne syndrome
C. Bloom syndrome
D. Werner syndrome
E. Xeroderma pigmentosum

A

►B

Patients with Rothmund thompson have early photosensitivity and poikilodermatous skin changes, juvenile cataracts, skeletal dysplasias, and a predisposition to osteosarcoma and skin cancer. Patients with Bloom syndrome have an increased risk for G I malignancies and lymphoma. Patients with xeroderma pigmentosum can develop ocular melanoma. Patients with Werner syndrome can develop thyroid and hematologic malignancies, sarcomas,and meningiomas. Cockayne syndrome patients have photosensitivity, short stature, premature aging, visual problems, neurologic deficits, but no internal malignancies.

How well did you know this?
1
Not at all
2
3
4
5
Perfectly
58
Q

58- A 6 month-old has a verrucous plaque on the mucosal surface of the lower lip. Skin biopsy is consistent with Riga-Fede disease. You should refer the patient to:

A. An ophthalmologist
B. A gastroenterologist
C. A neurologist
D. A hematologist
E. An otolarngologist

A

►C

Riga-Fede disease is a benign ulcerative granulomatous process that occurs in reaction to chronic, repetitive trauma of the oral mucosa by the teeth. Clinically, it appears as firm, verrucous plaques. It may be associated with an underlying developmental anomaly or underlying neurologic disorder.

How well did you know this?
1
Not at all
2
3
4
5
Perfectly
59
Q

59 -A 64-year old woman presents with scalloped erosions in the dermatome of the first branch of the trigeminal nerve. Biopsy would reveal which of the following histologic findings:

A. Asteroid Bodies
B. Caterpillar Bodies
C. Cowdry Type A Bodies
D. Donovan Bodies
E. Dutcher Bodies

A

►C

Varicella zoster virus is the causative agent in chicken pox and shingles, where recrudescence from latency in the dorsal root ganglion causes a dermatomal distribution of painful vesicles and erosions. Biopsy of herpes viruses reveals cells with multinucleation, margination of the chromatin, and Cowdry Type A bodies, which are intranuclear eosinophilic amorphous bodies surrounded by a clear halo.

How well did you know this?
1
Not at all
2
3
4
5
Perfectly
60
Q

60 -The most common autoimmune disease associated with hepatitis C is:

A. Autoimmune thyroiditis
B. Myasthenia gravis
C. Aplastic anemia
D. Lymphocytic sialadenitis
E. Rheumatoid arthritis

A

►A

The most common autoimmune disease seen in association with Hepatitis C is autoimmune thyroiditis.

How well did you know this?
1
Not at all
2
3
4
5
Perfectly
61
Q

61-All of the following are true regarding Cockayne Syndrome EXCEPT:

A. Inheritance is autosomal recessive
B. It is caused by an inability to repair cyclobutane dimers
C. It is associated with basal ganglia calcifications
D. It is associated with retinal pigment degeneration with a “salt and pepper” appearance
E. None of these answers are incorrect

A

►E

Cockayne Syndrome is an autosomal recessive disorder caused by a mutation of an unknown gene that results in the inability to repair cyclobutane dimers induced by UV exposure. Cutaneous findings include photosensitivity, “bird-headed” facies, and “Mickey Mouse” ears. Statements A-D are true regarding the syndrome.

How well did you know this?
1
Not at all
2
3
4
5
Perfectly
62
Q

62 -With regards to patients with diabetes mellitus and skin, which of the following is most accurate?

A. Diabetic dermopathy is most commonly expressed on the forearms and feet
B. Scleredema most often presents on the tibial surfaces
C. Scleredema is the most common skin manifestation of diabetes mellitus
D. Less than 5% of patients with diabetes mellitus have necrobiosis lipoidica
E. Less than 5% of patients with necrobiosis lipoidica have diabetes mellitus

A

►D

Patients with diabetes mellitus (DM) may develop thickened skin and tightened joints. Scleredema is occasionally seen in type II diabetics and presents with a peau d‟orange texture and appearance on the upper back. Perhaps as many as 20% of those with necrobiosis lipoidica diabeticorum(NLD) have diabetes, but only 0.3% to 3% of diabetics have NLD. The most common skin manifestation of DM is diabetic dermopathy, or skin hyperpigmentation, normally on tibial surfaces. Eruptive xanthomas, which are associated with high triglycerides are occasionally seen as are generalized or perforating granuloma annulare.

How well did you know this?
1
Not at all
2
3
4
5
Perfectly
63
Q

63- An 8-year-old boy presents with immediate burning of his skin on sun exposure. Biopsy of his dorsal hand is notable only for PAS-positive hyalinization of the vessel walls in the superficial papillary dermis. What is the most likely diagnosis?

A. Systemic lupus erythematosis
B. Atrophe blanche
C. Erythropoietic Protoporphyria
D. Actinic prurigo
E. Hydroa vacciniforme

A

►C

Erythropoietic Protoporphyria (EPP) is characterized by a burning sensation experienced immediately upon photoexposure. Biopsy reveals PAS-positive hyalinization of the superficial dermal vessel walls. Atrophe blance appears similar histologically but is not characterized by photosensitivity. The other disorders exhibit different histopathologic findings than described.

64
Q

64- Erythema gyratum repens is known to be associated with all of the following malignancies except:
A. Lung carcinoma
B. Breast carcinoma
C. Cervical carcinoma
D. Bladder carcinoma
E. Bone carcinoma

A

►E

Erythema gyratum repens presents more commonly in men than in women, and appears clinically as concentric erythematous rings with trailing scale on the trunk and proximal extremities. The skin is described as having a “wood grain” appearance. Skin findings often precede the diagnosis of an associated malignancy. Lung carcinoma is the most commonly associated malignancy, but it has also been reported in association with breast, cervical, bowel, prostate, gastric/bowel and bladder cancer.

65
Q

65 -A patient presents with a complaint of facial flushing that spreads to the neck and upper trunk. Review of systems reveals that the patient has occassional bouts of diarrhea, and intermittent bronchospasm. Laboratory testing reveals an elevated urine 5-hydroxyindole-acetic acid level. Which of the statements regarding this condition is NOT correct?

A. Chest and abdominal/pelvic CT scanning should be the next step in this patient’s evaluation
B. The patient may have an associated sclerodermoid-like eruption on examination
C. The patient likely has a neoplasm originating in the endocrine argentaffin cells
D. Treatment with cyproheptadine would be contraindicated
E. The patient likely has a neoplasm located in the GI tract

A

►D

This patient has carcinoid sydrome, which presents with facial flushing, diarrhea, and intermittent bronchospasm. Patients can also develop telangiectasia, pellagra-like, or sclerodermoid-like cutaneous findings. This syndrome is caused by a neoplasm originating in the endocrine argentaffin cells. 80-85% are found in the GI tract. Treatment is surgical removal of the tumor, or medical treatment with medications including somatostatin, methylsergide, cyproheptadine, beta-blockers, and phenothiazine derivatives.

66
Q

66- A patient comes to your office with extensive alopecia and melanocytic macules on the fingers. In the review of systems he states he has diarrhea and was told by his primary doctor that he has malabsorption symptoms. A previous colonoscopy revealed a few polyps which were removed. What kind of mutation does this patient have?

A. Autosomal recessive
B. Autosomal dominant
C. X-linked recessive
D. Sporadic
E. Mosaicism

A

►D

Patient has Cronkhite-Canada syndrome. This is a syndrome with a sporadic mutation. Patients develop GI symptoms(diarrhea, malabsorption, polyposis), weight loss and weakness. Later on they develop cutaneous changes such as melanocytic macules on the fingers, generalized hyperpigmentation and alopecia

67
Q

67- Which of the following is TRUE about diabetic-related skin disease?

A. Bullae are common on the thighs
B. Yellow skin affects the majority of diabetic patients
C. Diabetic dermopathy affects the upper back
D. Necrobiosis lipoidica affects approximately 20% of diabetics
E. Direct immunofluorenscence is usually negative in bullous diabeticorum

A

►E

Approximately 30% of diabetic patients have cutaneous manifestations of their disease. Diabetic dermopathy, or “shin spots” is the most common cutaneous association with diabetes. Patients are generally long-standing diabetics, and are associated with symmetrical brownish, atrophic plaques on the shins. Acanthosis nigricans is more common in black and Hispanic diabetics, and may be a component of the HAIRAN (hyperandrogen, insulin resistance, acanthosis nigricans)
syndrome. Hyperpigmented velvety plaques are most often found in the flexures. Waxy, thick skin and limited joint mobility are both related to poor glucose control. Scleredema diabeticorum, which most often causes thickened plaques of the upper back and neck most often affects type II diabetics. Necrobiosis lipoidica diabeticorum (NLD) is present in only 0.3 to 3% of diabetics, but perhaps 20% of patients with NLD have diabetes or glucose intolerance. Sharply-demarcated yellow-brown, telangiectatic plaques are found on the tibial surfaces. Ulceration may occur. Yellow skin may affect approximately 10% of diabetics and presents diffusely. Serum carotene levels may be elevated. Perforating disorders may be related, especially in the setting of renal disease. In the setting of hypertriglyceridemia, eruptive xanthomata may be present. The association between diabetes and granuloma annulare (GA) is controversial. If truly an association, the generalized and perforating forms of GA are most commonly associated. Infectious associations include candidiasis, which may present an angular cheilitis, chronic paronychia, interdigital or intertriginous disease. Cutaneous bacterial infections, including group A and B streptococci, pseudomonas aerugenosa and corynebacterium are more common. Dermatophyte infections are not more common, although tinea pedis may increase the risk for cellulitis in diabetic patients. Finally, rhinocerebral mucormycosis is a potential disasterous complication of uncontrolled hyperglycemia with ketoacidosis. Mortality rates can approach 35%.

68
Q

68- Hypothyroidism may result in all of the following cutaneous findings except:

A. Increase in the percentage of telogen hairs
B. Yellowish hue
C. Purpura
D. Madarosis
E. All of these answers are correct

A

►E

All of the above are non-specific cutaneous manifestations of hypothyroidism: an increase in percentage of telogen hairs, yellowish hue caused by carotenemia, purpura secondary to impaired wound healing, and loss of the lateral third of the eyeborw (madarosis).

69
Q

69- Regarding carcinoid syndrome, which of the following is TRUE?

A. Symptoms are caused by metastases to the skin
B. The most common location for the tumor is the rectum
C. Somatostatin is a preferred treatment
D. VMA is elevated in the urine
E. The tumors should not be removed

A

►C

Carcinoid syndrome is caused by a tumor originating in the endocrine argentaffin cells. In order of descending frequency, the most common locations are appendix > small bowel > rectum. Patients experience facial flushing that descends anatomically, diarrhea, hyperhidrosis and bronchial reactivity. A pellagra-like eruption may occur as well as telangiectasia and sclerodermoid changes. Patients experience symptoms once the disease metastasizes to the liver or if it did not involve the GI tract initially. Diagnosis can be made finding elevated 5hydroxyindolacetic acid (5-HIAA) in the urine. Nitrosonaphthol turns urine purple if 5-HIAA levels are significantly elevated, and can be used as a screening test. Bananas, tomatoes, plums, avocadoes and eggplant can increase levels of 5-HIAA. Treatment involves removal of the tumor along with medical therapy if appropriate. Agents used include somatostatin, methylsergide, cyproheptadine, and beta blockers. Vanillylmandelic acid (VMA) is an important urinary metabolic product of epinephrine and norepinephrine and is detected in the diagnosis of pheochromocytoma.

70
Q

70 -All of the following are true regarding Henoch-Schonlein Purpura EXCEPT:

A. It is an IgG mediated small vessel vasculitis
B. Direct immunofluorescence of lesional and peri-lesional skin will demonstrate C3 and fibrin deposits in small vessel wall
C. It is often preceded by an upper respiratory infection
D. It may be complicated by intussusception

E. It is self-resolving

A

►A

Henoch-Schonlein Purpura (HSP) is an IgA mediated small vessel vasculitis. Clinically, patients present with palpable purpura of the lower extremities and buttocks, GI vasculitis, and glomerulonephritis.

71
Q

71 -Which of the following statements regarding porphyrias is FALSE?

A. Fecal protoporphyrin is increased in variegate porphyria
B. Hepatoerythropoietic porphyria is the homozygous form of porphyria cutanea tarda
C. Acute intermittent porphyria has no skin findings
D. Coproporphyrinogen is elevated more than uroporphyrinogen in 24 hour urine samples in porphyria cutanea tarda
E. In erythropoietic protoporphyria, protoporphyrin IX absorbs in the Soret band

A

►D

Acute intermittent porphyria (AIP) is the second most common porphyria and is caused by a deficiency in porphobilinogen (PBG) deaminase, which is located in the cytosol. Patients suffer from colicky pain, paralysis and psychiatric disorders. There are no specific skin manifestations. PBG and aminolevulenic acid (ALA) are elevated in the urine. Attacks are precipitated by medications such as barbiturates, estrogen, griseofulvin, and sulfonamides as well as starvation, fever and infection. Treatment includes glucose loading and hematin infusion.

Congenital erythropoietic porphyria (CEP) or Günter‟s disease is caused by a defect in uroporphyrinogen III synthase, which is found in the cytosol. Patients are extremely photosensitive and erythema, blistering and scarring result. Patients present with red urine early in life along with hypertrichosis and red-stained teeth that fluoresce. Uroporphyrins (URO) are much high than coproporphyrins (COPRO) in the urine. URO is found in the red blood cells (rbcs) and COPRO is found in the stool. The rbcs display stable fluorescence.

Porphyria cutanea tarda (PCT) is the most common porphyria and is caused by a deficiency (usually sporadic) in uroporphyrinogen decarboxylase, which is found in the cytosol. Patients present with

photosensitivity and blistering of sun-exposed areas, especially the dorsal hands. Hypertrichosis and sclerodermoid changes may occur as well. Liver disease (hepatitis C or alcoholic cirrhosis) is often present and hemochromatosis may be associated. Urine may fluoresce pink or coral-red with Wood‟s lamp. URO>COPRO in the urine and low levels of COPRO are found in the stool. Treatments include phlebotomy, antimalarials, and therapy for liver disease if appropriate.

Hereditary coproporphyria (HCP) is caused by a deficiency in coproporphyrinogen oxidase, which is found in the mitochondria. One-third of patients are photosensitive, and patients suffer gastrointestinal and neurological symptoms similar to AIP. Urine COPRO is elevated only with attacks, and COPRO is present in the stool. Variegate porphyria is the result of decreased activity of protoporphyrinogen oxidase, which is present in the mitochondria. It combines the skin lesions of PCT with the systemic manifestations of AIP. Urine COPRO:URO is 1:1 or COPRO> URO to distinguish it from PCT, and PROTO is found in the stool. The plasma fluoresces at 626nm. Precipitators and treatments are similar to AIP.

Erythropoietic protoporphyria (EPP) is caused by ferrochetalase deficiency, which is present in the mitochondria. Patients experience immediate burning of the skin with sun exposure. Protoporphyrin IX, the only oxidized porphyrin in the heme pathway and absorbs in the Soret band (400-410nm). Patients have erythematous plaques in a photo-distribution. Urine porphyrins are normal. PROTO is found in the rbcs and the stool. Excessive porphyrins deposited in the liver lead to gallstones and cirrhosis. Beta carotene may helpful.

Hepatoerythropoietic porphyria (HEP) is essentially a homozygous form of PCT, with deficiency in uroporphyrinogen decarboxylase. It is clinically similar to CEP with red urine and hypertrichosis, vesicles and scarring of sun-exposed skin. URO is present in the urine and COPRO in the stool. PROTO is present in rbcs which distinguishes it from CEP, which was URO in rbcs.

72
Q

72 -Which of the following is FALSE regarding Fabry‟s disease?

A. It is inherited in an X-linked recessive fashion
B. It is associated with acral parasthesias
C. It is inherited in an X-linked dominant fashion
D. It may be associated with renal failure
E. The etiology is a defect in alpha-galactosidase A

A

►C

Fabry‟s disease is an X-linked lysosomal disorder that leads to excessive deposition of neutral glycosphingolipids in the vascular endothelium. The disorder is caused by a deficiency of alphagalactosidase-A leading to progressive endothelial accumulation of glycosphingolipids. This accumulation accounts for the associated clinical abnormalities of skin, eye, kidney, heart, brain, and peripheral nervous system. Acroparesthesias are a frequent presenting symptom of Fabry‟s disease. The primary cutaneous manifestation are angiokeratomata (angiokeratoma corporis diffusum), which are most common in a “bathing-trunk” distribution. Lens opacities and retinal and conjunctival vascular malformations may be found in the eyes. Patients may experience cardiac disease, stroke or renal failure.

73
Q

73- An end stage AIDS patient with tuberculosis presents with diffuse hyperpigmentation of both sun-exposed and unexposed areas. The palmar creases are markedly hyperpigmented and the patient is very ill. The most likely diagnosis is:

A. Argyria
B. Lymphoma
C. Tinea versicolor
D. Kaposi’s sarcoma
E. Addison disease

A

►E

Addison disease is caused by destruction of the adrenal glands by any cause. Tuberculosis used to be the primary cause, now the most common cause is auto-immune destruction. Other causes of adrenal gland destruction include; coccidiomycosis, cryptococcosis, histoplasmosis, sarcoidosis, metastatic tumor and amyloidosis.

74
Q

74- Which of the following is not a cause of a saddle nose deformity?

A. Relapsing polychondritis
B. Hypohidrotic ectodermal dysplasia
C. Congenital syphilis
D. Trichorhinophalangeal syndrome
E. Wegener’s granulomatosis

A

►D

Trichorhinopharyngeal syndrome is the only condition from the mentioned conditions that is not associated with saddle nose deformity.It is associated with sparse hair, a bulbous (pearshaped) nose and cone-shaped epiphyses.

75
Q

75- A patient infected with the hepatitis C virus is at highest risk for which of the following cutaneous manifestations?
A. Increased hyaluronic acid in dermis of lower and upper extremities
B. Lindsay‟s nails
C. Congenital hyperpigmentation of the retinal pigment (CHRPE)
D. Periorificial lentigines
E. Positive rheumatoid factor and decreased complement (C3)

A

►E

Patients with hepatitis C virus (HCV) may experience numerous cutaneous manifestations of their internal disease. Cryoglobulinemia (CG) may result in small vessel leukocytoclastic vasculitis. Types of CG include: type I, which is monoclonal IgM; type II, which is polyclonal IgG and monoclonal IgM; type III, which is polyclonal IgG and polyclonal IgM. Mixed CG is the most commonly associated type of CG with HCV. Patients often have a positive rhematoid factor and have C3 complement decreased. Other diseases associated with HCV include porphyria cutanea tarda, lichen planus and polyarteritis nodosa. Thyroid-related dermopathy results in increased hyaluronic acid in the dermis. Lindsay’s nails, or half and half nails, are associated with renal disease. Congenital hyperpigmentation of the retinal pigment (CHRPE) is seen in Gardner’s syndrome. Peutz-Jeghers, among other conditions presents with periorificial lentigines.

76
Q

76- A 24-year-old woman presents with painful sores involving the bilateral cheeks. Biopsy reveals a dense lichenoid infiltrate with Civatte body formation. What is the most likely associated systemic disease:

A. Diabetes mellitus
B. Hepatitis C Infection
C. Hypothyroidism
D. Chronic kidney disease
E. Castleman‟s disease

A

►B

Lichen planus of the oral mucosa is most highly associated with Hepatitis C Infection. Biopsies of lichen planus on the oral mucosa may exhibit plasma cells and parakeratosis, unlike cutaneous lichen planus. The other disorders are not as closely related to oral lichen planus.

77
Q

77 -Regarding eruptive xanthomas, which of the following is true?

A. They occur in the setting of familial hyperlipidemia types I, IV, and V
B. They occur in the setting of familial hyperlipidemia types II and III
C. Triglyceride levels are usually below 500mg/dl
D. They are most commonly found on the eyelids
E. They are not related to alcohol consumption

A

►A

Eruptive xanthomas generally occur in patients with triglyceride levels of 2000mg/dl or greater. Associations include poorly-controlled diabetes mellitus, retinoids, estrogens, excessive alcohol consumption (leading to pancreatitis) and familial hyperlipidemias types I, IV and V. Clinically, they appear as crops of firm, non-tender yellowish papules with an erythematous border. Most commonly, they occur on the extensor surfaces, but they can be diffuse. A reduction in triglycerides and/or tight glucose control usually results in a reduction in the number of lesions.

78
Q

78- Patients with this syndrome are at increased risk for developing Lhermite-Duclos disease:

A. Bourneville’s Disease
B. Nail-Patella Syndrome
C. MEN Type IIA
D. Fabry’s Disease
E. Cowden’s Syndrome

A

►E

Patients with Cowden’s Syndrome (multiple hamartoma syndrome) are at increased risk for Lhermite Duclos disease (dysplastic gangliocytoma of the cerebellum).
c

79
Q

79- The patient demonstrated in figure would have:

A. LDL receptor deficiency
B. Decreased chylomicrons
C. Elevated triglycerides
D. Decreased LDL
E. Decreased triglycerides

A

►C

This patient has eruptive xanthomas, a condition which is classically associated with elevated serum triglycerides.

80
Q

80- Which of the following statements about necrolytic migratory erythema is TRUE?

A. There is a low incidence of metastasis of the offending tumor at the time of diagnosis
B. Acanthosis and parakeratosis are found on routine histology
C. Serum glucagon levels are usually normal
D. Vacuolar changes are normally found on routine histology

E. The offending tumor originates from endocrine argentaffin cells

A

►B

Necrolytic migratory erythema or glucagonoma syndrome is clinically characterized by periorificial and acral erythema, vesicles, pustules and erosions. A circinate pattern is often seen. Glossitis and cheilitis are features as well. The underlying tumor is an alpha-2 glucagon producing islet cell pancreatic carcinoma, which is metastatic at the time of diagnosis in the majority of cases. Histopathological findings include dyskeratotic keratinocytes in the stratum granulosum, acanthosis and parakeratosis. Serum glucagon levels are elevated. The tumors must be resected if feasible, and intravenous somatostatin and amino acids have been used for treatment.

81
Q

81- Which of the following is NOT associated with Cronkhite-Canada syndrome?

A. Lentigines
B. Adenomatous gastrointestinal polyps
C. Fibrocystic breast disease
D. Onycholysis
E. Weight loss

A

►C

Cronkhite-Canada syndrome (CCS) is a rare, sporadically occurring, non-inherited disorder characterized by generalized gastrointestinal polyps, cutaneous pigmentation, alopecia, and onychodystrophy. The pathogenesis is unknown. The GI polyps are hamartomatous and malignant degeneration is unusual. Cutaneous signs include circumscribed lentiginous hyperpigmentation, alopecia and nail dystrophy. Often, symptoms appear in the sequence of gastrointestinal symptoms, weight loss, weakness, edema, and then cutaneous changes after weeks or months. CCS is a progressive disease, and often carriers a poor prognosis primarily because of difficulties with fluid and electrolyte management. Fibrocystic breast disease is associated with Cowden”s disease (multiple hamartoma syndrome).

82
Q

82- A 34-year-old woman presents with dell-shaped papules of the bilateral cheeks. Biopsy of a representative lesion reveals clear cell syringoma. Which is the most likely associated systemic disease:

A. Diabetes mellitus
B. Hepatitis C Infection
C. Hypothyroidism
D. Chronic kidney disease
E. Castleman‟s disease

A

►A

Clear cell syringomas are associated with diabetes mellitus. Eruptive syringomas may arise in patients with skin type V and VI and in patients with Down‟s syndrome. The other disorders are not associated with syringomas.

83
Q

83- A patient with end stage renal disease complains of some itchy spots on his back. Examination reveals flesh-colored papules with a central keratotic core. Acquired perforating dermatosis is more commonly seen in end stage renal disease due to what underlying disorder?

A. Diabetic nephropathy
B. Lupus nephritis
C. Mesangial glomerulonephropathy
D. IgA nephropathy
E. Medication-related renal failure

A

►A

Acquired perforating dermatosis is the transepidermal elimination of altered dermal substances. The etiology is unknown, but may be related to micro-deposits of calcium with subsequent evacuation through the epidermis. It is more commonly seen in darker skin types and in patients with ESRD secondary to diabetic nephropathy.

84
Q

84 -Which of the following is NOT normally associated with dermatomyositis?

A. Ovarian cancer in women
B. Psoriasiform dermatitis of the scalp
C. Elevated aldolase levels
D. Cuticular dystrophy
E. Testicular cancer in men

A

►E

Dermatomyositis is characterized by proximal muscle weakness and a constellation of cutaneous findings. Skin findings include periorbital edema and poikiloderma (heliotrope), Gottron‟s sign and papules, nail fold telangiectasia, cuticular dystrophy, poikiloderma in a shawl distribution and photosensitivity. Psoriasiform dermatitis and scaling of the palms and soles may occur-NOT on the scalp! Children more often display calcinosis cutis and vasculitis. Within the first 3 years of diagnosis there is a higher likelihood of malignancy. Women are at risk for ovarian and breast carcinoma, and men are at risk for gastric carcinoma and lymphoma. Diagnostic findings include elevated creatine kinase (CK) and aldolase, inflammatory muscle biopsy changes or abnormal muscle MRI, and abnormal EMG. Treatment includes corticosteroids, methotrexate and other immunosuppressive medications.

85
Q

85- Which of the following is NOT true regarding Cushing’s Syndrome?

A. Urine cortisol levels are elevated
B. Corticotropin is suppressed with the administration of dexamethasone
C. It may be associated with an underlying oat cell lung carcinoma
D. It may be associated with hypertension and hypokalemia
E. May present with hyperpigmentation and facial plethora

A

►B

In Cushing’s Syndrome, corticotropin is NOT suppressed with the administration of dexamethasone. The other statements are true.

86
Q

86- The shoulder pad sign has been described in which disease?

A. Dermatomyositis
B. Cushings disease
C. Systemic amyloidosis
D. Nephrogenic fibosing dermopathy
E. Systemic lupus erythematosis

A

►C

The shoulder pad sign has been described in the setting of late systemic amyloidosis and is due to the direct deposition of amyloid in the deltoid muscles.

87
Q

87- Paraneoplastic pemphigus is associated with all of the following underlying malignancies EXCEPT:
A. Non-Hodgkins lymphoma
B. Lung carcinoma
C. Chronic lymphocytic leukemia
D. Thymoma
E. Castleman’s tumor

A

►B
Paraneoplastic pemphigus has been associated with Non-Hodgkins lymphoma, chronic lymphocytic leukemia, thymoma, Castleman’s tumor, and sarcoma. It has not been associated with lung carcinoma. Treatment includes management of the underlying malignancy, as well as prednisone or other immunosuppressive agents.

88
Q

88- Which of the following is true regarding cutaneous associations with hepatitis C virus (HCV) infection?

A. Erosive mucosal lichen planus has a weaker association with HCV than does cutaneous lichen planus

B. Pruritus in the setting of chronic HCV infection is generally correlated with elevated bile salt levels in the setting of liver failure
C. Polyarteritis nodosa is associated with HCV infection but not hepatitis B virus infection
D. HCV-related porphyria cutanea tarda may be caused by decompartmentalization of iron stores and resultant oxidation of uroporphyrinogen decarboxylase
E. Cutaneous reactions to HCV treatment are less common with interferon/ribavirin combination treatment than with treatment with interferon alone

A

►D

Erosive mucosal variant of lichen planus has the strongest association with HCV. The pathogenesis of pruritus in the setting of chronic HCV infection may be related to elevated bile salt levels in the setting of liver failure, but there is not always a direct correlation between serum bile salt level and degree of pruritus. Polyarteritis nodosa is associated with both HCV infection and hepatitis B virus infection. Statement D is correct. Cutaneous reactions to HCV treatment are more common with interferon/ribavirin combination treatment than with treatment with interferon alone.

89
Q

89 -A patient presents with diffuse waxy keratoderma of the palms and soles as well as oral hairy leukoplakia. Which of the following statements regarding this condition is NOT true?

A. This condition may be associated with esophageal carcinoma
B. This condition is autosomal recessive
C. Family members should be advised to undergo cancer screening
D. Features may include squamous cell carcinomas arising from keratodermic skin
E. None of these answers are correct (all are true)

A

►B

Howel-Evans syndrome is an autosomal dominant disorder that presents with diffuse waxy keratoderma of the palms and soles, as well as oral leukoplakia and squamous cell carcinoma arising from keratodermic skin. It is associated with esophageal carcinoma; cancer screening in family members is advised.

90
Q

90 -Which of the following laboratory abnormalities is most common in patients with cholesterol emboli?

A. Neutrophilia
B. Elevated amylase
C. Hypercalcemia
D. Eosinophilia
E. Anemia

A

►D

Eosinophilia develops within 3 days of embolization in 70-80% of patients and may remain elevated for up to 1 month. Patients with cholesterol emboli typically have an elevated ESR and C- reactive protein as well. Leukocytosis can also be seen in up to half of patients.

91
Q

Regarding paraneoplastic pemphigus, which of the following is TRUE?

A. Desmoplakin, one of the molecular antigens, has a molecular weight of 190kd
B. Metastatic squamous cell carcinoma of the skin is a common cause
C. GranularC3 deposition is found at the dermoepidermal junction on direct immunofluorescence
D. Monkey esophagus is the preferred substrate for indirect immunofluorescence
E. Non-Hodgkins lymphoma is rarely associated

A

►C

Paraneoplastic pemphigus (PNP) is characterized by painful stomatitis, which is extremely
resistant to therapy. Cutaneous lesions are variable and can include flaccid or tense bullae, targetoid lesions or lichenoid eruptions. PNP is associated with the following neoplasms (in descending order of frequency): non-Hodgkins lymphoma, chronic lymphocytic leukemia, Castleman‟s disease and malignant and benign thymus tumors. Histopathologic findings include suprabasal acantholysis, dyskeratosis and vacuolar interface with a lichenoid infiltrate. Direct immunofluorescence of perilesional skin demonstrates intercellular and granular IgG and granular C3 at the dermoepidermal junction. The preferred substrate for indirect immunofluorescence is rat bladder and intercellular IgG is found. Target antigens include desmoplakin (250kD), envoplakin (210kD),

bullous pemphigoid antigen-1 (230kD), periplakin (190kD), and desmogleins 1 and 3. Many patients succumb to the underlying cancer. Treatment requires management of the malignancy and immunosuppressive agents.

92
Q

92- Which of the following is an autosomal recessive disorder characterized by pili torti and deafness?

A. Waardenburg Syndrome
B. Sjorgen-Larsson Syndrome
C. Refsum Disease
D. Bjornstad’s Syndrome
E. Cockayne Syndrome

A

►D

Waardenburg Syndrome is an autosomal dominant disorder. Sjorgen-Larsson Syndrome is an autosomal recessive disorder characterized by a triad of ichthyosis, di- or tetraplegia, mental retardation. Refsum Disease is an autosomal recessive disorder whose features include deafness, mild ichthyosis in adulthood, and accentuated palmar creases. Bjornstad’s Syndrome is an autosomal recessive disorder characterized by pili torti and deafness. Cockayne Syndrome is an autosomal recessive disorder whose features include photosensitivity, bird-headed facies, and “Mickey-Mouse” ears, as well as deafness.

93
Q

Which of the following is characteristic of diabetic skin?

A. Approximately 20% of diabetics have necrobiosis lipoidica diabeticorum (NLD)
B. The level of cleavage in bullous diabeticorum is subcorneal
C. Candida tropicalis is the most common cause of angular cheilitis
D. There is a well-established association between deep granuloma annulare (GA) and diabetes
E. Yellow skin may occur in up to 10% of diabetics

A

►E

Perhaps 0.3 to 3% of diabetics have NLD, whereas approximately 20% of NLD patients have diabetes or glucose intolerance. The split in bullous diabeticorum is normally either intraepidermal or subepidermal. C. albicans is the most common cause of diabetes-related yeast infections. The association between GA and diabetes is controversial, but if the two are related, generalized and perforating GA have been implicated. Yellow skin may occur in up to 10% of diabetic patients and is characterized by diffuse yellow-orange skin. Half of these patients have elevated serum carotene levels. The suggested is cause is elevated consumption of yellow fruits and vegetables in the setting of impaired hepatic metabolism of carotene and subsequent non-
enzymatic glycosylation of dermal collagen.

94
Q

94- A patient develops cold-exacerbated dusky acral plaques consistent with pernio. The systemic condition most likely to manifest such lesions is:

A. Lupus Erythematosus
B. Dermatomyositis
C. Acquired Immunodeficiency Syndrome
D. Hepatitis C
E. Diabetes Mellitus
c

A

►A

Chilblain lupus is a rare manifestation of lupus erythematosus. Lesions resemble common pernio clinically, but should have histological findings consistent with lupus, such as interface changes. These lesions may not resolve upon warming and often respond poorly to other lupus treatments such as antimalarials.
c

95
Q

95 -The most common location of the lesions in nephrogenic fibrosing dermopathy is:

A. Face
B. Palms and soles
C. Lower extremities
D. Back
E. Chest

A

►C

The most common location for lesions of nephrogenic fibrosis dermopathy is the lower extremities.

96
Q

96 -Which of the following syndromes necessitates a work-up for colon cancer?

A. Turcot
B. Birt-Hogg-Dube
C. LAMB
D. Muckle Wells
E. Gardner

A

►A

Turcot syndrome is a variant of HNPCC-hereditary nonpolyposis colorectal cancer, Lynch syndrome with the homozygous mutations in MSH2, MSH6, MLH1, PMS2 as opposed to heterozygous mutaitons in HNPCC. It is characterized by polyposis and brain cancer. Patients may also exhibit other clinical features of HNPCC syndrome. Both patients with Lynch Syndrome and Turcot Syndrome necessitate a colon cancer work-up.
c

97
Q

97- Which step is rate-limiting in the synthesis of protoporphyrinogen?

A. Aminolevulinic acid synthase
B. Aminolevulinic acid dehydratase
C. Porphobillinogen deaminase
D. Coproporphyrinogen oxidase
E. Ferrochelatase

A

►A

The rate-limiting step in the heme synthesis pathway is the synthesis of delta aminolevulanic acid from glycine and succinyl CoA via aminolevulinic acid synthase. This step takes place in the

mitochondria. Porphobillinogen deaminase is defective in acute intermittent porphyria, coproporphyrinogen oxidase in in hereditary coproporphyria and protoporphyrinogen oxidase in variegate porphyria.
c

98
Q

98- Carotenemia can be a manifestation of:

A. Porphyria
B. Hypothyroidism
C. Pretibial myxedema
D. Grave”s disease
E. Amyloidosis

A

►B

Reduced matabolism of beta-carotene in the diet, can result in yellowing of the skin in hypothyroidism. Treatment of porphyria with beta-carotene can result in carotenemia. Pretibial myxedema, Grave”s disease and amyloidosis do not result in carotenemia.
c

99
Q

99- Which of the following is (are) characteristic of basal cell nevus syndrome?

A. Autosomal recessive inheritance
B. NEMO gene mutation
C. Tram-track calcifications
D. Colobomas
E. Telangiectasias

A

►D

Basal cell nevus syndrome (Gorlin syndrome) is caused by an autosomal dominant mutation in PTCH gene that encodes PTC protein involved in sonic hedge hog pathway. This mutation leads to loss of inhibition of smoothened (SMO) leading to an increased expression of other genes. Cutaneous findings include nevoid basal cell carcinomas, milial cysts, epidermoid cysts and palmar- plantar pits. Neurological findings include calcifications of the falx cerebri, agenesis of the corpus

callosum, mental retardation, medulloblastoma and spina bifida. Ocular findings include blindness, cataracts, colobomas and strabismus. Other findings include odontogenic cysts of the jaw, frontal bossing, bifid ribs, and pectus deformity. NEMO gene mutations are found in incontinentia pigmenti. Tram-track calfications are seen Sturge-Weber syndrome. Telangiectasias are seen in ataxia-telangiectasia among others.

100
Q

100- A patient with gastric cancer develops acanthosis nigricans and a sudden eruption of numerous warty stuck-on papules on the trunk. What other finding may be seen?

A. Numerous wart-like lesions on the dorsal hands and wrists
B. Follicular spicules on the nose
C. Carpal tunnel syndrome
D. Periorbital pupura
E. Migratory thrombophlebitis

A

►A

Florid cutaneous papillomatosis is the rapid onset of numerous wart-like lesions on the backs of the hands and wrists that is associated with pruritus, acanthosis nigricans, and the sign of LeserTrelat. All reported cases are associated with internal malignancy, most commonly gastric carcinoma. Follicular papules on the nose are seen in multiple myeloma. Carpal tunnel syndrome and periorbital purpura can be seen in primary systemic amyloidosis with multiple myeloma. migratory thrombophlebitis is associated with pancreatic cancer.
c

101
Q

101- An uncommon complication of treatment with potassium iodide is:

A. Acute generalized exanthematous pustulosis
B. Erythema nodosum
C. Wolff-Chiakoff effect
D. Exacerbation of lichen planus
E. Hyperhidrosis

A

►C

An uncommon complication of treatment with potassium iodide is the Wolff-Chiakoff effect.

102
Q

102- Which of the following is true about blue rubber bleb nevus syndrome?

A. typically appears in adulthood
B. the venous malformations regress with time
C. characterized by compressible blue nodules 0.1 to 5 cm in size.
D. the nodules are pruritic
E. the gastrointestinal lesions are typically located in the stomach

A

►C

Blue rubber bleb nevus is associated with soft and compressible blue nodules 0.1 to 5 cm in size. The cutaneous lesions typically appear in childhood, and they do not regress with time. The venous malformations are mainly asymptomatic but can be painful when thrombosis occurs. The venous malformations also involve the gastrointestinal tract, most commonly the small bowel, where they are friable and can bleed.

103
Q

103- A 14-year-old girl presents with a 20-pound weight loss along with painful oral erosions and severe stomatitis. Which is the most likely associated systemic disorder:

A. Small cell lung cancer
B. Hepatitis C Infection
C. Hypothyroidism
D. Chronic kidney disease
E. Castleman‟s disease

A

►E

Castleman‟s tumor is the most common cause of paraneoplastic pemphigus in children. Paraneoplastic pemphigus presents with painful stomatitis and varied skin lesions. Indirect immunofluorescence with rat bladder substrate shows intercellular IgG.

104
Q

104- All of the following are true regarding the condition pictured EXCEPT:

A. Perilesional direct immunofluroescence shows granular IgA in the dermal papillae and at the dermoepidermal junction
B. Only 20% of patients have a gluten-sensitive enteropathy
C. It is associated with HLA-DQ2, HLA-DR3, and HLA-B8
D. It is associated with Hashimoto’s thyroiditis
E. Cutaneous findings are due to autoantibodies to epidermal transglutaminase

A

►B

All of the statements regarding dermatitis herpetiforms, or “Duhring’s Disease,” are true except for statement B. Virtually all DH patients have gluten-sensitive enteropathy, although only 20% of them have symptoms (such as diarrhea, steatorrhea, weight loss, bloating, and malabsorption).

105
Q

105- A 64-year old man develops yellowish periorbital plaques that occasionally ulcerate and heal with scarring. What is the most likely associated lab finding?

A. Monoclonal gammopathy
B. Positive antinuclear antibodies
C. Elevated creatinine
D. Elevated triglycerides
E. Increased thyroid stimulating hormone

A

►A

The patient has necrobiotic xanthogranuloma. Lesions develop periorbitally and in flexural areas, often extend deeply into the dermis or subcutis, and heal with scarring. These lesions are typically associated with a monoclonal gammopathy, usually IgG. Patients with necrobiotic xanthogranuloma rarely develop myeloma. Leukopenia and hepatosplenomegaly are also often seen.

106
Q

106- Which of the following cutaneous findings are characteristic for Vohwinkles Syndrome?

A. Honeycombed diffuse palmoplantar keratoderma, pseudoainhum with autoamputation, star-shaped keratosis over knuckles, nail dystrophy, and alopecia
B. Dense depigmented lusterless hair, pili torti, doughy skin, diffuse cutaneous hypopigmentation
C. Transient erythroderma at birth, palmoplantar keratoderma, follicular hyperkeratosis, scarring alopecia, dystrophic nails
D. Rapidly progressive alopecia of all hair-bearing areas, onycholysis, onychoschizia, onychomadesis, hyperpigmented macules on extremities
E. Oral papillomatosis, palmoplantar keratoses, acral keratoses, lipomas, hemangiomas, scrotal tongue

A

►A

Vohwinkles Syndrome is an autosomal dominant disorder caused by a mutation of the GJB2 gene which encodes Connexin 26. Clinical features include deafness, as well as the cutaneous findings described in choice A.

107
Q

107- Which of the following is NOT true regarding calciphylaxis?

A. Patients with proximally-located lesions have a better prognosis than those with acral lesions
B. May be treated with parathyroidectomy
C. May be present with retiform purpura
D. Histologic findings include medial calcification and intimal hyperplasia of small arteries and arteriole
E. Has an associated mortality of 60-80%

A

►A

Distribution of lesions in calciphylaxis may predict prognosis; those with acral lesions have a better outcome than those with proximally located lesions.

108
Q

108- Features of Cushing disease include all of the following EXCEPT?

A. Facial plethora
B. Striae
C. Hypertension
D. Suppression of corticotropin occurs with administration of dexamethasone
E. Hirsutism

A

►D

Cushing syndrome is caused by excess levels of either exogenously administrated glucocorticoids or endogenous overproduction of cortisol. The syndrome is most commonly caused by the therapeutic administration of exogenous glucocorticoids. The term “Cushing’s disease” is reserved for Cushing’s syndrome that is caused by excessive secretion of adrenocorticotropin hormone (ACTH) by a pituitary tumor, usually an ad enoma. Other endogenous sources include an adrenal tumor and other malignancies (i.e. oat cell carcinoma of the lung). Patients develop moon facies, facial plethora, supraclavicular fat pads, buffalo hump, truncal obesity, and striae. In addition, proximal muscle weakness, easy bruising, weight gain, hirsutism, and, in children, growth retardation may occur. Systemic sequelae include hypertension, osteopenia, diabetes mellitus, and impaired immune function. Excess endogenous or exogenous glucocorticoids can result in a leukocytosis and hypokalemic metabolic acidosis. Urine cortisol levels 3 to 4 times normal are diagnostic. The dexamethasone suppression test is a valuable diagnostic tool. For this study, 1mg is given at 11pm and the 8am cortisol level is tested. A level of >10 ìg/dL is diagnostic. In patients with Cushing‟s disease, the dexamethasone does not suppress corticotropin. Corticotropin-releasing hormone (CRH) stimulation test is also used to distinguish patients with pituitary adenomas and those with ectopic ACTH syndrome or cortisol-secreting adrenal tumors.

109
Q

109- What is the treatment of choice for porphyria cutanea tarda?

A. Phlebotomy
B. Antimalarials
C. Erythropoietin
D. Oral iron supplementation
E. Naproxen

A

►A

Porphyria cutanea tarda (PCT) is caused by a deficiency in the enzyme uroporphyrinogen decarboxylase. Phlebotomy is the treatment of choice and may improve enzymatic activity by removing iron, an inhibitor of the enzyme. Antimalarials and erythropoietin are alternative therapies. Oral iron supplementation may worsen PCT. Naproxen is a common cause of pseudoporphyria.

110
Q

110- Which of the following statements about multiple endocrine neoplasia (MEN) syndromes is FALSE?

A. MEN I is associated with angiofibromas and collagenomas
B. MEN IIa is related to a defect in RET proto-oncogene
C. MEN IIa and IIb are both associated with medullary thyroid carcinoma
D. MEN I, IIa and IIb are all inherited in an autosomal dominant fashion
E. MEN IIa is associated with multiple mucosal neuromas
c

A

►E

Multiple endocrine neoplasia (MEN) syndromes are divided into types I, IIa and IIb. All are autosomal dominantly inherited. Type I is associated with mutations in MENI; type IIa and IIb involve mutations of RET, which encodes a tyrosine kinase receptor. Type I has the following cutaneous features: facial angiofibromas, collagenomas, lipomas, hypopigmented macules and café- au-lait macules. Type IIa displays macular amyloidosis, while IIb features multiple mucosal neuromas. Systemic features of t ype I include peptic ulcer disease (as part of Zollinger-Ellison syndrome), parathyroid hyperplasia or adenoma, pancreatic tumors and pituitary neoplasia. Type IIa is characterized by Zollinger-Ellison syndrome, medullary thyroid carcinoma, pheochromocytoma, and parathyroid neoplasia. Type IIb displays marfanoid habitus, Hirschsprung disease, medulloblastoma, medullary thyroid carcinoma, pheochromocytoma and ocular neuromas.

111
Q

-Tripe palms are a cutaneous manifestation associated with which of the following malignancies?

A. Renal carcinoma
B. Lung carcinoma
C. Prostate carcinoma
D. Colon carcinoma
E. Pancreatic carcinoma

A

►B

Tripe palms are rugose thickening of the palms which are nearly always associated with internal malignancy. When tripe palms are present without other cutaneous findings, the most likely internal malignancy is lung cancer. Tripe palms in association with acanthosis nigricans is most likely to be associated with gastric cancer.

112
Q

112- The clinical findings in the figure are associated with which disorder?

A. Grave”s disease
B. Myasthenia gravis
C. Dermatomyositis
D. Sarcoidosis
E. Rheumatoid Arthritis

A

►A

This slide shows pretibial myxedema in a patient with Grave‟s disease.

Pretibial myxedema is a form of diffuse mucinosis in which there is an accumulation of excess glycosaminoglycans in the dermis and subcutis of the skin. Glycosaminoglycans, also called mucopolysaccharides, are complex carbohydrates that are important for tissue hydration and lubrication. The main glycosaminoglycan in pretibial myxedema is hyaluronic acid, which is made by cells called the fibroblasts. Pretibial myxedema is also known as localized myxedema, thyroid dermopathy, and infiltrative dermopathy. It is most commonly seen on the shins (pretibial areas) and is characterized by swelling and lumpiness of the lower legs.

113
Q

113- A 7 year old boy is seen for “acne”. Hypopigmented patches are seen on the trunk and flesh colored papules are seen around the nails. Biopsy of one of the papules of the face reveals an angiofibroma. A review of his chart reveals that he takes medications for seizures. What is the most common neoplasm associated with the most likely disorder?

A. Renal angiomyolipoma
B. Renal cell carcinoma
C. Gastric carcinoma
D. Transitional cell carcinoma
E. Prostate carcinoma

A

►A

Tuberous sclerosis is an autosomal dominant disorder with seizures, mental retardation, and characteristic skin findings, including hypopigmented macules, facial angiofibromas, periungual fibromas, and collagenomas. The most common associated tumor is renal angiomyolipoma. Rarely, renal cell carcinoma can develop.

114
Q

114- What is the most common internal cause of intractable pruritus?

A. Hepatitis C
B. Hypothyroidism
C. Hyperthyroidism
D. Chronic renal failure
E. Internal malignancy

A

►D

Chronic renal failure is the most common internal systemic cause of pruritus. Up to 49% of patients with chronic renal failure have pruritus. Other systemic causes of pruritus include liver disease, hepatitis C, hypo and hyperthryoidism, iron deficiency anemia, polycyhtemia vera, Hodkin’s lymphoma, leukemia, carcinoid, internal malignancy, AIDS, and internal parasites.

115
Q

115- Which of the following may be an early cutaneous manifestation of Sipple syndrome (multiple endocrine neoplasia type 2A)?

A. mucosal neuromas
B. Koenen tumor
C. mandibular osteomas
D. multiple pilomatricomas
E. cutaneous amyloidosis

A

►E

Sipple syndrome (multiple endocrine neoplasia type 2A) is an autosomal dominant disorder characterized by the development of pheochromocytomas, medullary thyroid carcinomas, and hyperparathyroidism. The associated mutation is found in the RET proto-oncogene. An early cutaneous manifestation includes the development of lichen or macular amyloidosis (both keratin- derived cutaneous amyloidoses) often localized to interscapular patches, which may be pruritic (similar to notalgia paresthetica). The latter arises during teenage years and may be the earliest presenting sign of this disease.

116
Q

116- Mutations in the STK11 gene encoding a serine threonine kinase are seen in:

A. Muir-Torre Syndrome
B. Bannayan-Riley-Ruvalcaba Syndrome
C. Birt-Hogg-Dubbe Syndrome
D. Cronkhite-Canada Syndrome
E. Peutz-Jeghers Syndrome

A

►E

Peutz-Jeghers Syndrome is an autosomal dominant syndrome. 50% of patients have mutations in the STK11 gene which encodes a serine threonine kinase. It presents with periorificial and mucosal lentigines beginning in infancy, as well as hamartomatous intestinal polyps with intussusception and bleeding; gallbladder, pancreatic, breast, ovarian, and testicular cancer.

117
Q

117- Which of the following is true regarding treatment of inflammatory dermatoses with potassium iodide?

A. The Wolff-Chaikoff effect must be considered
B. Binding of excess organic iodide in the thryoid gland may occur
C. Thyroid hormone synthesis may be inhibited
D. None of the answers are correct
E. All of these answers are correct

A

►E

The Wolff-Chaikoff effect is described as the binding of excess organic iodide in the thyroid gland with resultant inhibition of thyroid hormone synthesis. This can occur in the setting of patients with erythema nodosum (or other inflammatory dermatoses) being treated with potassium iodide.

118
Q

118- A 36-year-old woman presents with infiltrative, verrucous plaques of the bilateral lower extremities in association with a 20-pound weight loss. Biopsy reveals increased mucin deposition in the dermis without increased cellularity. Which is the most likely associated systemic disease:

A. Diabetes mellitus
B. Hepatitis C Infection
C. Hyperthyroidism
D. Chronic kidney disease
E. Castleman‟s disease

A

►C

Grave‟s disease is associated with pretibial myxedema. Pathology reveals diffuse increased mucin in the dermis, without increased fibroblasts as seen in scleromyxedema and nephrogenic systemic fibrosis. The other disorder are not associated with pretibial myxedema.

119
Q

119- A 2 year old child is evaluated for suspected diagnosis of neurofibromatosis. Which of the following diagnostic findings is typically absent on exam in this age group?

A. neurofibromas
B. macrocephaly
C. pigmented iris hamartomas
D. seizures
E. cafe au lait macules

A

►A

Cutaneous neurofibromas typically appear after puberty. The absence of neurofibromas in young children does not rule out this diagnosis. Pigmented iris hamartomas are more likely finding in young children than neurofibromas.

120
Q

120- A 56-year-old female presents with longitudinal grooving of the nail plate of her index finger. Examination reveals a 4mm spongy subcutaneous nodule just distal to the DIP joint. Biopsy reveals a cyst with no true lining. What is the most common associated systemic disease:

A. Hypothyroidism
B. Bronchiolitis Obliterans Pneumonia (BOOP)
C. Hypertrophic Cardiomyopathy
D. Sporotrichosis
E. Osteoarthritis

A

►E

Digital myxoid cysts are composed of mucin contained in a cystic structure without a true lining. They may have connections to the joint space, especially in patient‟s with osteoarthritis, who are

predisposed to developing myxoid cysts. Historically, tuberculosis infection has also been associated with myxoid cysts but there is little current support for this relationship.
c

121
Q

121- A patient that has congenital erythropoietic porphyria and has red urine with severe photosensitivity. The patient also has redness, swelling, blistering in the sun-exposed areas with resultant scarring. The defect is in the:

A. Uroporphyrinogen III synthase homozygous defect
B. Protoporphyrinogen oxidase deficiency
C. Decreased fecal coproporphyrin
D. Elevated urinary porphobilinogen
E. Coproporphyrinogen oxidase deficiency

A

►A

The defect in congenital erythropoietic porphyria is in the uroporphyrinogen III synthase homozygous defect. It is present after birth with erythrodontia, hyperthrichosis, growth retardation, hemolytic anemia, and increased bone features.

122
Q

122- Each of the following demonstrates a vasculitis except:

A. Granuloma faciale
B. Henoch-Schoenlein purpura
C. Type 1 cryoglobulinemia
D. Wegener‟s granulomatosis
E. Leukocytoclastic vasculitis

A

►C

In type I cryoglobulinemia, monoclonal IgG or IgM cryoglobulins are found often in association with lymphoma, leukemia, Waldenstrom‟s macroglobulinemia, or multiple myeloma. On histopathology, type I cryoglobulinemia is characterized by the deposition of precipitated amorphous cryoglobulins on the endothelium and throughout the vessel wall. The precipitates stain

with PAS stain. An inflammatory infiltrate is typically lacking in contrast to mixed cryoglobulinemia (which shows a leukocytoclastic vascultitis)

123
Q

123- A patient on hemodialysis presents with indurated plaques having a peau d‟orange texture on the bilateral lower legs. Which of the following statements is TRUE?

A. The diagnostic histopathological findings include acanthosis and hyperkeratosis
B. Serum protein electrophoresis should be performed
C. Dapsone will likely be effective treatment
D. The face is usually is affected
E. The palms and soles are usually affected

A

►B

This patient likely has nephrogenic fibrosing dermopathy (NFD). NFD is an acquired, idiopathic disorder that occurs in renal disease patients. It resembles scleroderma or eosinophilic fasciitis clinically and scleromyxedema histopathologically. Large areas of indurated skin with fibrotic nodules and plaques develop. The extremities are most commonly involved, followed by the trunk. The face, palms and soles are almost never involved. Histopathologically, NFD displays a proliferation of dermal fibroblasts and dendritic cells, thickened collagen bundles, increased elastic fibers, and mucin deposition. Serum protein electrophoresis and immunoelectrophoresis results are negative unlike scleromyxedema, and may be helpful to distinguish the two diseases. NFD is usually a chronic, progressive condition, and favorable responses to medical intervention are anecdotal.

124
Q

124- All of the following are true regarding incontinentia pigmenti EXCEPT:

A. It is caused by a mutation in the NEMO gene
B. It is an X-linked recessive disorder
C. Inflammation and blistering may be followed by hyperkeratotic, verrucous lesions
D. It is associated with cerebellar ataxia
E. It is associated with coloboma and retinal detachment

A

►B

Incontinentia pigmenti is an X-linked dominant disorder caused by a mutation in the NEMO gene. Females only present at birth with linear lesions of inflammation and blistering (stage 1), followed by hyperkeratotic verrucous areas (stage 2), hyperpigmentation (stage 3), and then atrophy (stage 4). Systemic findings include psychomotor retardation, microcephaly, seizures, cerebellar ataxia, coloboma, and retinal detachment.

125
Q

125- Hepatitis C infection is associated with:

A. Pityriasi rubra pilaris
B. Mixed cryoglobulinemia
C. Pityriasis rosea
D. Kaposi’s sarcoma
E. Oral hairy leukoplakia

A

►B

Cutaneous manifestation associated with hepatitis C include necrolytic acral erythema, porphyria cutanea tarda, lichen planus, polyarteritis nodosa and mixed cryoglobulinemia. Mixed cryoglobulinemias is a systemic vasculitis with variable manifestations including palpable purpura, arthralgias and weakness.
c

126
Q

126 -What is the unique laboratory finding in stool in porphyria cutanea tarda (PCT) and hepatoerythropoietic porphyria (HEP)?

A. elevated coproporphyrins
B. elevated isocoproporphyrins
C. decreased isocoproporphyrins
D. elevated protoporphyrins
E. elevated uroporphyrins

A

►B

PCT and HEP are associated with elevated isocoproporphyrins in the stool. Congenital erythropoietic porphyria, hereditary coproporphyria is associated with increased coproporphyria in the stool, and elevated stool proporphyria is seen in variegate porphyria.

127
Q

127- A 12 year old boy is referred for evaluation of a rash in his groin. It has become progressively worse over the preceding three years. Discrete erythematous papules are seen on the lower anterior abdomen, upper thighs, and buttocks. Biopsy of a representative papule reveals an angiokeratoma. An evaluation by a nephrologist reveals proteinura. A diagnosis of Fabry's disease is made. Renal failure in this syndrome is related to accumulation of what substance in the kidney?

A. Ceramide trihexoside
B. Galabiosylceramide
C. Hyaluornic acid
D. Mucopolysaccharide
E. Alpha-galactosidase

A

►A

Fabry’s disease is an x-linked recessive disorder characterized by angiokeratomas in a bathing trunk distribution, hypohidrosis, corneal opacities, and acral paresthesias. The diagnosis is confirmed by decreased levels of alph-galactosidase in white blood cells, serum, and fibroblasts. Renal failure is due to accumulation of ceramide trihexoside aka Globotriaosylceramide.

128
Q

128- All of the following are true regarding the condition pictured EXCEPT:

A. Men with this condition may be at increased risk for lymphoma
B. Associated calcinosis cutis may be seen in pediatric patients
C. Age-appropriate cancer screening is recommended
D. May be associated with a psoriasiform scalp dermatitis
E. Elevated aldolase is more specific than CPK

A

►E

All of the statements regarding dermatomyositis are true except for statement E. Elevated muscle enzymes are found in dermatomyositis, but elevated CPK is more specific than aldolase. Diagnosis is also established with muscle biopsy showing evidence of inflammation and abnormal EMG.

129
Q

129- In patients with mixed cryoglobulinema associated with hepatitis C, the most likely laboratory abnormality is:

A. Elevated rheumatoid factor
B. + ANA
C. + p-ANCA
D. Decreased cryoglobulins
E. Elevated hematocrit
c

A

►A

In patients with mixed cryoglobulinemia, the most likely laboratory abnormality among the options listed is an elevated rheumatoid factor.

130
Q

130- Regarding eruptive xanthomata, which of the following is TRUE?

A. They are associated with type II and III hyperlipidemias
B. They are associated with calcium channel blockers
C. There is no association with ethanol consumption
D. They are associated with type I, IV, and V hyperlipidemias
E. They favor the flexor surfaces of the extremities

A

►D

Eruptive xanthomata appear as erythematous to yellow papule, from1 to 4mm in diameter, generally distributed over the extensor arms, hands, and buttocks. They are present in response to either primary or secondary hypertriglyceridemia. Primary causes are discussed below. Secondary causes include obesity, diabetes, excessive alcohol consumption, estrogens, and systemic retinoids. Type I hyperlipidemia is caused by lipoprotein lipase deficiency and patients have elevated levels of

triglycerides (TG) and chylomicrons (chylo). Type I may b e associated with eruptive xanthomata, lipemia retinalis, abdominal pain, pancreatitis, and hepatosplenomegaly. Type IIA is heterozygous for apolipoprotein B deficiency, and results in increased low-density lipoproteins (LDL). Patients may show tendinous or tuberous xanthomata and are at high risk for coronary artery disease (CAD) and stroke. Type IIB is homozygous for apolipoprotein B deficiency and patients have elevated LDL, very low-density lipoprotein (VLDL) and TG. Intertriginous and tuberous xanthomata in addition to advanced atherosclerosis, CAD and stroke at an early age are associated with type IIB. Type III is caused by apolipoprotein E deficiency resulting in elevated intermediate-density lipoproteins (IDL) and TG, and displays palmar/plantar, tendinous and tuberous xanthomata. This disease is associated with diabetes, gout, CAD and stroke. Type IV disease is characterized by increased TG and VLDL and patients may have eruptive xanthomata along with CAD, diabetes and stroke. Type V is caused by an apolipoprotein C2 defect resulting in increased TG, chylo, and VLDL. Eruptive xanthomata, diabetes, hepatosplenomegaly, lipemia retinalis and pancreatitis are associated with type V.

131
Q

131 -All of the following statements regarding cryoglobulinemia are true EXCEPT:

A. Type I is composed of monoclonal IgG and polyclonal IgM
B. Type II is composed of polyclonal IgG and monoclonal IgM
C. Type III is composed of polyclonal IgG and polyclonal IgM
D. 80% of cases of mixed cryoglobulinemia are associated with Hepatitis C infection
E. None of these answers are correct (all statements are true)

A

►A

Type I is composed of monoclonal immunoglobulins.

132
Q

132- Which of the following statements regarding multiple endocrine neoplasia syndromes is true?

A. MEN Type IIB is also known as Sipple’s Syndrome
B. MEN Type IIA is also known as Wermer’s Syndrome
C. Multiple mucosal neuromas are seen in association with MEN Type IIA
D. Lichen or macular amyloidosis is seen in association with MEN Type IIA

E. Patients with MEN Type IIB are at increased risk for developing follicular thyroid carcinoma
c

A

►D

MEN Type I is also known as Wermer’s Syndrome. MEN Type IIA is also known as Sipple’s Syndrome. Mucosal neuromas are seen in association with MEN Type IIB. Statement D is correct. Patients with MEN Type IIB are at increased risk for developing medullary thyroid carcinoma.

133
Q

133- A 64-year-old woman presents with lichenified keratotic plaques on the bilateral lower extremities. Biopsy reveals suppurative inflammation and collagen extending from the dermis through the epidermis. Which is the most likely associated systemic disease:

A. Congestive Heart Failure
B. Hepatitis C Infection
C. Hypothyroidism
D. Chronic kidney disease
E. Castleman‟s disease

A

►D

Chronic kidney disease is associated with perforating disorders. These disorder are characterized by pruritus and a biopsy revealing collagen and inflammatory cells broaching the epidermis. The other disorders are not closely associated with perforating diseases.

134
Q

134 -A patient presents with episodic flushing of the face and neck, abdominal pain, wheezing, cough, and diarrhea. Carcinoid syndrome is suspected. Which of the following tests would confirm the diagnosis?

A. Change of urine to green color upon addition of nitrosonaphthol
B. Elevated urinary 5-hydroxyindolacetic acid
C. Elevated 24-hour urine norepinephrine
D. Decreased 24-hour urine vanilylmandelic acid

E. Elevated serum tryptase

A

►B

The urine in a patient with carcinoid syndrome features high levels of 5 -hydroxyindolacetic acid and changes color to purple upon addition of nitrosonaphthol. Elevated 24-hour urine norepinephrine or vanilylmandelic acid are seen in pheochromocytoma. Mastocytosis features high levels of tryptase.

135
Q

135- What is the most common malignancy associated with this condition at this location?

A. Breast cancer
B. Gastric cancer
C. Thyroid cancer
D. Pancreatic cancer
E. Melanoma

A

►B

Malignant acanthosis nigricans usually presents with sudden onset and is rapidly progressive. It may be associated with diffuse keratodermas of the palms and soles or eruptive seborrheic keratoses.

136
Q

136- This autosomal dominant condition is characterized by trichodiscomas, fibroepithelial polyps, and fibrofolliculomas:

A. POEMS syndrome

B. Rombo syndrome
C. Birt-Hogg-Dube syndrome
D. Proteus syndrome
E. Cowden’s disease
c

A

►C

Birt-Hogg-Dube is an autosomal dominant condition characterized by trichodiscomas, fibroepilethial polyps and fibrofolliculomas.

137
Q

137- Bilateral diagonal earlobe creases may be associated with which of the following systemic diseases?

A. Gout
B. Chronic renal insufficiency
C. Interstitial pulmonary fibrosis
D. Hepatobiliary cirrhosis
E. Atherosclerotic coronary artery disease

A

►E

Several studies have indicated that bilateral diagonal earlobe creases (known as “Frank‟s sign”) may serve as a clinical marker for underl ying coronary artery disease independent of age, particularly in persons younger than 40 years old.

138
Q

138- A 58 year old female with lifelong Type 1 diabetes mellitus has end-stage renal failure. What is the most common cutaneous manifestation of end stage renal disease?

A. Pruritus
B. Nephrogenic systemic fibrosis
C. Hyperpigmentation
D. Acquired perforating dermatosis
E. Skin pallor

A

►A

There are may cutaneous manifestations of end-stage renal disease. Chronic anemia can cause skin pallor. Deposition of carotenoids can give the skin a yellowish hue. Photo-distributed hyperpigmentation and ecchymoses are also commonly seen. The most common cutaneous manifestation, however, is pruritus. Other cutaneous findings include metastatic calcification, acquired perforating dermatosis, and nephrogenic systemic fibrosis.

139
Q

139 -One might see all of the following laboratory and clinical abnormalities in cryoglobulinemia associated with hepatitis C virus infection EXCEPT:

A. Elevated liver function tests
B. Positive rheumatoid factor
C. Elevated C3 levels
D. Acrocyanosis
E. Urticarial plaques

A

►C

Laboratory abnormalities of HCV include an elevation of liver enzymes, positive rheumatoid factor (70-90%), and DEPRESSED C3 levels. Classical clinical presentation includes palpable purpura, arthralgias, and glomerulonephritis. Livedo reticularis, hemorrhagic bullae, acrocyanosis, and urticarial plaques can also be found.

140
Q

When metastases to the skin occur from a thyroid malignancy, they are usually due to:

A. Medullary carcinoma
B. Papillary adenocarcinoma
C. Follicular carcinoma
D. Anaplastic carcinoma
E. Cutaneous metastases have not been reported in association with thyroid malignancies

A

►B

Metastases to the skin from a thyroid malignancy are rare, but most reported cases occur with papillary adenocarcinoma.

141
Q

141 -The presence of antibodies to c-ANCA is characteristically seen in patients with which disease?

A. Ulverative colitis
B. Churg-Strauss disease
C. Polyarteritis nodosa
D. Wegener”s granulornatosis
E. Nodular vasculitis

A

►D

Wegener”s granulomatosis is a necrotizing granulomatous disorder that most commonly affects the upper and lower respiratory tracts, kidneys, and eye. Cytoplasmic pattern ant ineutrophil cytoplasmic autoantibody (anti -proteinase-3) is often positive in this disease. Churg-Strauss is associated with a positive p-ANCA.
c

142
Q

142- Which type of porphyria is the autosomal recessive form of porphyria cutanea tarda?

A. Erythropoietic protoporphyria (EPP)
B. Congenital erythropoietic porphyria (CEP)
C. Hepatoerythropoietic porphyria (HEP)
D. Variegate Porphyria (VP)
E. Acute Intermitent Porphyria (AIP)

A

►C

HEP results from deficient, but not absent, activity of uroporphyrinogen decarboxylase (UROD). HEP is the recessive form of familial PCT. It manifests during infancy or early childhood as photosensitivity, skin fragility in sun-exposed areas, pink urine, erythrodontia, and hypertrichosis.

143
Q

143 -A 70-year old male develops hyperpigmented velvety plaques on his lips, dorsal hands and feet, and in his axilla. What underlying malignancy is most commonly associated with this finding?

A. Gastric Carcinoma
B. Lung Carcinoma
C. Ovarian Carcinoma
D. Lymphoma
E. Breast Carcinoma

A

►A

Acanthosis Nigricans can be associated with obesity, insulin resistance, Crouzon”s syndrome, congenital lipodystrophy, and internal malignancy. 90% of related malignancies are tumors within the abdominal cavity. Adenocarcinoma of the stomach is the most common. Malignancy related acanthosis nigricans often occurs in the setting of weight loss, helping distinguish it from other associated diseases.

144
Q

144- All of the following are true regarding calciphylaxis except:

A. Seen in end-stage renal disease
B. Vascular mural calcification occurs late in the process
C. High morality rate is due to sepsis
D. Association with hyperparathyroidism
E. Association with an elevated calcium: phosphate product

A

►B

Calciphylaxis is a rare, life threatening disorder that is associated with end stage renal disease. Clinically, patients develop a livedoid purpura and enlarging, tender, indurated subcutaneous plaque typically on the legs or abdomen. These plaques are subject to ischemia, skin necrosis, and secondary infection, sepsis, and death. Histologic examination of calciphylaxis is characterized by the triad of small vessel mural calcification, extravascular calcification, and vascular thrombosis. It appears that vascular mural calcification is an early and essential process in the development of calciphylaxis.

145
Q

145 - A complication seen in women of childbearing age with the condition in the figure is:
A. Atrial septal defect
B. Pulmonary lymphangioleiomyoma
C. Gastrointestinal bleeding
D. Lisch nodules
E. Pulmonary emboli

A

►B

This picture demonstrates a shagreen patch in a patient with Tuberous sclerosis. A complication occurring in female patients of child-bearing age is pulmonary lymphangioleiomyoma.

146
Q

146- Porphyria cutanea tarda may be associated with all of the following except:

A. Hepatitis C virus infection
B. Alcohol
C. Estrogens
D. Polyhalogenated hydrocarbons
E. Inherited deficiency of uroporphyrinogen III synthase

A

►E

A homozygous defect in uroporphyrinogen III synthase in a cause of congenital erythropoietic porphyria, not porphyria cutanea tarda (PCT). PCT may be caused by sporadic or familial deficiency in uroporphyrinogen decarboxylase, or by any of the other causes listed above.

147
Q

147- Which of the following is true regarding nephrogenic fibrosing dermopathy?

A. Has a rapidly progressive but reversible course
B. Is associated with a paraproteinemia
C. Is associated with peripheral eosinophilia
D. May be associated with antiphospholipid antibodies
E. Is associated with a dramatic increase in dermal mucin

A

►D

Nephrogenic fibrosing dermopathy has an indolent course and treatment is usually not satisfactory. It is not associated with a paraproteinemia or peripheral eosinophilia. Some patients have been reported to have antiphospholipid antibodies. Histopathology demonstrates a minimal to slight increase in dermal mucin.

148
Q

148- A patient is diagnosed with acute intermittent porphyria with periodic attacks of colic, paralyses and psychiatric disorders. All of the following can precipitate attacks except:

A. Barbiturates
B. Estrogen
C. Griseofulvin
D. Sulfonamides
E. Tetracyclines

A

►E

Patients with acute intermittent porphyria have a deficiency in porphobilinogen deaminase. Attacks are precipitated by barbiturates, estrogen, griseofulvin, and sulfonamides. Only 10% of those with genetic defect develop the disease but all at risk for primary liver cancer.

149
Q

149- A 50-year old woman with a history of spontaneous pneumothorax develops multiple firm, skin colored lesions on her face and neck over a period of several years. This patient should have periodic surveillance for the development of:
A. Renal Cell Carcinoma
B. Gastric Carcinoma
C. Breast Carcinoma
D. Ovarian Carcinoma
E. Lung Carcinoma

A

►A

This patient has Birt-Hogg-Dube syndrome, an autosomal dominant defect of the BHD gene that encodes folliculin. This syndrome is characterized by the development of trichodiscomas, fibrofolliculomas, and acrochordons starting around age 30. The patients are also at risk for the development of spontaneous pneumothorax and renal cell carcinoma.

150
Q

150- The organism that causes this infection shown in the figure is:

A. T. tonsuran
B. C. immitis
C. C. albicans
D. P. aeruginoas
E. T. verrucosum

A

►C

The organism that causes erosio interdigitalis blastomycetica is candida albicans.

151
Q

151- What is the most common primary site of a carcinoid tumor?

A. Liver
B. Stomach
C. Appendix
D. Duodenum
E. Ileum

A

►E

The most common site of carcinoid tumor is the ileum. Clinical symptoms of episodic flush ing, abdominal pain, diarrhea, wheezing, and a pellagra-like dermatosis (due to shunting of tryptophan to serotonin) etc present with metastases to the liver. In a report from the SEER database of 11,427 carcinoid cases treated between 1973 and 1997, the majority were located in the gastrointestinal (GI) tract (55 percent) and bronchopulmonary system (30 percent). Within the GI tract, most carcinoids arose in the small intestine (45 percent, most commonly in the ileum), followed by rectum (20 percent), appendix (16 percent), colon (11 percent), and stomach (7 percent)

152
Q

152 -Which of the following is more commonly associated with ulcerative colitis as compared to Crohn‟s disease?

A. Pyoderma gangrenosum
B. Oral lesions
C. Polyarteritis nodosa
D. Fistulae
E. Metastatic lesions

A

►A

Crohn‟s disease (CD) and ulcerative colitis (UC) may both have gastrointestinal tract (GIT) and cutaneous manifestations. CD lesions can involve any area of the GIT (from the lips to the anus), with patchy disease normally present, whereas UC is limited to the colon and rectum and tends to be continuous. Oral disease, fissures and fistulae, metastatic disease to the skin, and
polyarteritis nodosa are more commonly associated with CD. Conversely, erythema nodosum, pyoderma gangrenosum and pyoderma vegetans tend to be more commonly associated with UC.

153
Q

153 -A 63-year-old male develops small, non-tender, violaceous papules on his dorsal hands, face, ears, and trunk. The patient also develops a destructive arthropathy with finger deformities. What percentage of patients with this condition has an associated malignancy?

A. 20-25%
B. 5-10%
C. 65-70%
D. 50-55%
E. 85-90%

A

►A

This patient has developed multicentric reticulohistiocytosis. Histopathology of the skin lesions will display multinucleated oncocytic giant cells with eosinophilic ground glass cytoplasm. Half of these patients will develop a mutilating arthritis. 20-25% of patients with multicentric reticulohistiocytosis will develop an associated malignancy. The malignancy is usually a carcinoma, but there is no predominant type.
c

154
Q

154 -A young African American patient presents with anemia and spontaneously appearing leg ulcers over both lateral and medial malleoli. The most likely diagnosis is:

A. Atherosclerotic disease
B. Sickle cell anemia
C. Factitial dermatitis
D. Lupus erythematosus
E. Trauma

A

►B

Sickle cell anemia should be considered most likely in any young African American with spontaneous leg ulcers. The ulcers are more common in people with severe anemia.

155
Q

155 -A 6-year-old girl presents with immediate burning of his skin on sun exposure and is ultimately diagnosed with Erythropoietic Protoporphyria (EPP). Which internal organ is most likely to be affected?

A. Heart
B. Lungs
C. Liver
D. Kidneys
E. Pancreas

A

►C

Erythropoietic Protoporphyria (EPP) is characterized by a burning sensation experienced immediately upon photoexposure. Biopsy reveals PAS-positive hyalinization of the superficial dermal vessel walls. This disorder is due to ferrochelatase deficiency. Excess porphyrins are deposited in the liver, which may become cirrhotic.